The Health Care System Mid Term

Pataasin ang iyong marka sa homework at exams ngayon gamit ang Quizwiz!

Medical Home

The quality features of primary health care delivery in settings such as a physician office or community health center

Demand

The quality of health care purchased; Is driven by the prices prevailing in the free market

Accountability

The responsibility of clinicians and patients, respectively, for the provision and receipt of efficient and quality health care services

Health Care

The treatment of illness and the maintenance of health

Bioterrorism

The use of chemical, biological, and nuclear agents to cause harm to relatively large civilian populations

Gatekeeping

The use of primary care physicians to coordinate health care services needed by enrollees in a managed care plan

Technological Imperative

The use of technology without cost considerations, especially when the benefits to be derived from the use of technology are small compared to the costs

Quality-Adjusted Life Year (QALY)

The value of 1 year of high-quality life, used as a measure of health benefit

Holistic Health

The well-being of every aspect of what makes a person whole and complete

Doctoral Nursing Degrees

These include in the Doctor of Nursing Practice (DNP), Doctor of Nursing Science (DNS), and Doctor of Philosophy in Nursing (PhD)

Uninsured

Those without private or public health insurance coverage

From the standpoint of health insurance, what were the main accomplishments of the Affordable Care Act?

Through the Affordable care act more than 20 million people got insured. There were lesser uninsured people in America compared to the 2014, there were less number of people left uninsured as the affordable care act costed less than what American people had to pay originally. The American people have more options to choose from hospitals and doctors compared to past where people had little choice of hospitals and doctors. This law has made people financially stable as they have to spend lesser on medical insurance than before. Through the affordable care act the US government was able to levy more tax through medical insurance on rich and financial freedom to poor, bridging the gap between poor and rich. The Affordable Care act had given the Small Business Health Options program to small business owners to provide health and dental insurance to their employees. They can get this insurance through a private insurance company at a lower cost.

Title XVIII

Title 18 of the Social Security Amendment of 1965; Refers to the medicare program

Title XIX

Title 19 of the Social security amendment of 1965; Refers to the medicaid program

Medicaid

Title XIX (Social Security Act, 1965) to make health care available to those people with less than the minimum income who do not qualify for Medicare; An extension of the Kerr-Mills program of federal matching funds to the states based on each states financial needs

Medicare

Title XVIII (Social Security Act, 1965) to provide a measure of health coverage to all Social Security recipients, Part A finances hospital insurance and partial nursing home coverage for the elderly through Social Security, and Part B covers physicians' bills through government-subsidized insurance

What are some of the main requirements for Medicare certification of a hospice program?

To receive Medicare certification, a hospice must meet these basic conditions: -Provide physician certification that the patient's prognosis is for a life expectancy of 6 months or less -Make nursing services, physician services, and drugs and biologics available on a 24-hour basis -Provide nursing services under the supervision of a registered nurse -Make arrangements for inpatient care when necessary -Provide social services by a qualified social worker under the direction of a physician -Make counseling services available to both the patient and the family including bereavement support after the patient's death -Provide needed medications, medical supplies, and equipment for pain management and palliation -Provide physical, occupational, and speech therapy services when necessary -Provide home health aide and homemaker services when needed

Provide a brief description of the roles and responsibilities of health services administrators

Top-level health services administrators provide leadership and strategic direction, work closely with governing boards, and are responsible for an organization's long-term success. They are responsible for the operational, clinical, and financial outcomes of their entire organization. Middle-level administrators may have leadership roles for major service centers such as outpatient, surgical, and nursing services, or they may be departmental mangers in charge of single departments. Their jobs involve major planning and coordinating functions, organizing human and physical resources, directing and supervising, operational and financial controls, and decision making. They often have direct responsibility for implementing changes, creating efficiencies, and developing new procedures with respect to changes in the health care delivery system. Entry-level administrators may function as assistants to middle-level managers and may supervise a small number of operatives

National Health Expenditures

Total amount spent for all health services and supplies and health-related research and construction activities consumed in the United States during a calendar year

Medical Tourism

Travel abroad to receive elective, non-emergency medical care

Outliers

Unusual cases that call for additional reimbursement under a payment method; These are atypical cases requiring an exceptionally long inpatient stay or exceptionally high costs compared to the overall distribution of cases

Asynchronous Technology

Use of store-and-forward technology that allows the users to review the information at a later time

Telemedicine

Use of telecommunications technology that enables physicians to conduct two-way, interactive video consultations or transmit digital images, such as X-rays and magnetic resonance imaging results, to other sites

Health promotion and disease prevention may require both behavioral modification and therapeutic intervention. Discuss

Various avenues can be used in motivating individuals to alter behaviors that may contribute to disease, disability, or death. Behavior can be modified through educational programs and incentives directed at specific high-risk populations. Therapeutic interventions fall into three areas of preventive effort: primary prevention (refers to activities undertaken to reduce the probability a disease will develop in the future), secondary prevention (refers to early detection and treatment of disease), and tertiary prevention (refers to interventions that could prevent complications from chronic conditions and prevent further illness, injury, or disability)

Globalization

Various forms of cross-border economic activities driven by the global exchange of information, production of goods and services more economically in developing countries, and increased interdependence of mature and emerging world economies

Urgent Care Centers

Walk-in clinics that are generally open to see patients after normal business hours in the evenings and weekends, and for which patients do not have to make an appointment

Cost Shifting

When the amount of reimbursement from some payer becomes inadequate or when uncompensated services are rendered without payment from some source, cost shifting is a mechanism used to make up for revenue shortfalls. In cost shifting, providers charge extra to payers who do not exercise strict cost controls

Universal Coverage

Would enable all citizens and legal residents to have health insurance

Balance Bill

is used by Medicare to allow physicians to recoup their differences that is not paid by insurance but Medicaid does not allow balance billing and has limited participation from physicians

Provide a brief overview of how technology influences the quality of medical care and quality of life

quality of medical care: patient is believing and feeling-technology provides more precise diagnosis, offer quicker and more complete cures, and reduce risks quality of life: adding life to years-technology improves ability to lead a more normal life, and relieve pain and suffering

Utilization

split into crude measures (Access to primary care services=# of persons in a given population who visited a primary care provider in a given year/ size of the population or Utilization of primary care=#of primary care visits by people in a given population in a given year/size of the population), specific measures (Utilization of targeted services=# of people in a specific targeted population using special services or visits/size of the targeted population group or Utilization of specific inpatient services=# of inpatient days/size of population), and measures of institution-specific (Average daily census=Total # of inpatient days in a given time period/# of days in the same time period or Occupancy rate= Total # of inpatient days in a given time period / Total # of available beds during the same time period aka average daily census/Total # of beds in facility or Average length of stay= Total # of inpatient days during a given time period/ Total # of patients served during the same time period)

How did the Supreme Court's ruling on the ACA affect Medicaid? How did the ACA affect the Medicaid program in terms of coverage and cost?

· ACA required all states to cover residents under the age 65 but this mandate was struck down by the Supreme Court thus giving states the option to either expand or not expand their Medicaid programs without any penalty from the federal government, 31 states did expand Medicaid while the 19 other states did not, and Medicaid expansion allowed a significant number of low-income people gain health coverage. Under federal law, access to emergency department does not require insurance coverage but post-ACA era Medicaid paid use of hospital emergency departments increased by 27%. Under the ACA, Medicaid payments to providers were brought up to Medicare levels only temporarily. Churning occurred and forced people to find new coverage through the government sponsored exchanges. Medicaid created a two-tier health care system with one branch serving the poor and the other branch reserved for the nonpoor

What is adverse selection? What are its consequences?

· Adverse Selection= occurs when high risk individuals enroll in health insurance plans to greater numbers compared to people who are healthy. Conversely, a health plan may face lower enrollments of young and healthy people thus premiums are raised for everyone

What is meant by health care financing in its broad sense? How does financing affect the health care delivery system?

· As its central role, health services financing pays for health insurance premiums. Providers generally rely on the patients' insurance to get paid for the services they deliver. The various methods used to determine how much providers should be paid (i.e. reimbursement) for their services are also closely intertwined with the broad financing function · To a large extent, financing determines who has access to health care and who does not although many have access to charitable care · The demand for health care greatly influences its financing. Health insurance increases the demand for covered services. Increased demand means greater utilization of health services given adequate supply · Financing also exerts powerful influences on supply-side factors such as how much health care is produced in the private sector. Health care services and technology proliferate when services are covered by insurance. New services and technologies emerge, and new models of organization may form. Conversely, when reimbursement is cut, supply of health care services can also be curtailed · Demand-side factors including reimbursement typically guide health services managers in evaluating the type and extent of services to offer. The amount of reimbursement needed to recoup capital costs over time also heavily influences decisions such as acquisition of new equipment, renovation, or expansion of facilities, and launching of new services · Financing can influence the supply and distribution of health care professionals · Financing affects the total health care expenditures incurred by a health care delivery system directly and indirectly eventually

Describe the major types of health service professionals (physicians, nurses, dentists, pharmacists, physician assistants, nurse practitioners, certified nurse midwives) including their roles, training, practice requirements, and practice settings

· Certified nurse mid-wives (CNMs) are RNs with additional training from a nurse-midwifery program. In areas such as maternal and fetal procedures, maternity and child nursing, and patient assessment, CNMs deliver babies, provide family planning education, manage gynecologic and obstetric care, and can substitute for obstetricians/gynecologists in prenatal and postnatal care. They are certified by the American College of Nurse-Midwives to provide care for normal expectant mothers. They refer abnormal or high-risk patients to obstetricians or jointly manage the care of such patients.

Provide brief descriptions of clinical information systems, administrative information systems, and decision support systems in healthcare delivery

· Clinical Information Systems(involves the organized processing, storage, and retrieval of information to support patient care delivery such as electronic medical records and computer provider order entry (CPOE)), Administrative Information Systems (assists health care staff in carrying out financial and administrative support activities such as payroll, patient accounting, billing, materials management, budgeting, and cost control and office automation), and Decision Support Systems (provides information and analytical tools to support managerial and clinical decision making such as forecasting patient volume, projecting staffing requirements, scheduling patients to optimize utilization of patient care and surgical facilities, and improve clinical decision making)

What is the relationship between reimbursement cuts and cost shifting? How do hospitals react in different markets to cuts in reimbursement?

· Cost-shifting= mechanism used to make up for revenue shortfalls due to the amount of reimbursement from some payer being inadequate or when uncompensated services are rendered without payment from some source, providers charge extra to payers who do not exercise strict cost controls Studies on cost-shifting by hospitals show hospitals in less competitive markets raised prices to private insurers when faced with shortfalls between Medicare payments and their projected costs (Robinson, 2011)/ Conversely, in competitive markets, hospitals focus on cutting costs when faced with reimbursement shortfalls from public payers. The expansion of coverage under ACA will be paid for in part by reducing payments to hospitals and other providers. In response, hospitals and other providers have consolidated, eroding competition. Large providers devise new ways to shift costs

What is the difference between experience rating and community rating?

· Experience rating= based on a group's own medical claims experience, premiums differ under this method because different groups have different risks, and high-risk groups are charged higher premiums because of the expectation of incurring high utilization of medical care services · Community rating=spreads the risk among members of a larger population, premiums based on the utilization experience of the entire population covered by the same type of health insurance, same rate applies to everyone regardless of age, gender, occupation, or any other indicator of health risk

What are the major methods of reimbursement for outpatient services?

· Fee for service is old but still in use, based on the assumption that health care is provided in a set of identifiable and individually distinct units of services, each service is separately itemized on one bill, providers started to balance bill when insurance started to restrict payment, and main problem is providers have incentive to deliver additional services that are not always essential thus increasing their incomes by increasing the volume of services · Fee for service essentially pays for unbundled services. A bundled fee includes a number of related services in one price, reduces the incentive for providing nonessential services, and reduce health care spending without significantly affecting quality of care · Resource Based Relative Value Scale (RBRVS) =reimbursement mechanism to pay physicians according to a relative value assigned to each physician service, incorporates relative value units based on the time, skill, and intensity (physician work) it takes to provide a service which reflect resource input such as time, effort, and expertise to deliver a service and separate RVUs for the cost of practice, malpractice insurance, and geographic cost variations · Medicare Physician Fee Schedule (MPFS)= a standard dollar amount called a conversion factor is used and a sustainable growth rate factor is applied to create a price list for physician services based on which individual payments are made when physicians file their claims · MCOs have 3 approaches to payment: Preferred-Provider Approach (contract with certain preferred providers and negotiates discounts off the charges to establish fee schedules), Capitation (provider paid a set monthly fee per enrollee at a fixed rate regardless of how often enrollees receive medical services from the provider, and Salary combined with productivity-related bonuses (used by some MCOs that employ their own physicians) · Medicare implemented Outpatient Prospective Payment System to pay for services provided by hospital outpatient department under the ambulatory payment classification. All services within an APC have the same payment rate. New technologies covered as well by APC. Expensive drugs and biologicals have separate APCs. Reimbursement rates adjusted for factors such as geographic variations in wages and also for outpatient services delivered by certain cancer centers and children's hospitals. APC reimbursement is in the form of a bundled rate includes services such as anesthesia, certain drugs, supplies, and recovery room charges in a packaged price established by Medicare. They now pay for facility services such as nursing, recovery care, anesthetics, drugs, and other supplies, in ambulatory surgery centers, and physician services are reimbursed separately under the physician fee schedule based on RBRVS

How has technology affected access to medical care?

· Geography is an important factor in access to technology. If technology is not physically available to a patient population living in remote areas, access is limited. Geographic access to many technologies can be improved by providing mobile equipment or by employing new communications technologies to allow remote access to centralized equipment and specialized personnel. Mobile equipment can be transported to rural and remote areas making it accessible to those populations. the US is the primary exporter of medical tech worldwide, helping people get care in other parts of the world

Discuss the general concept of insurance and its general principles. Describe the various types of private health insurance options, pointing out the differences among them

· Insurance= a mechanism for protection against risk that is its primary purpose · Risk= refers to the possibility of a substantial financial loss from an event of which the probability of occurrence is relatively small · Even when the risk is small, people buy insurance to protect their assets against catastrophic loss · Insurer= the insuring agency that assumes risk and known as underwriter · Underwriting= a systematic technique for evaluating, selecting (or rejecting), classifying, and rating risks · Four fundamental principals underlie the concept of insurance: 1) Risk is unpredictable for the individual insured, 2) Risk can be predicted with a reasonable degree of accuracy for a large group of a population, 3) Insurance provides a mechanism for transferring or shifting risk from the individual to the group through the pooling of resources, and 4) All members of the insured group share actual losses on some equitable basis · Private health insurance=" voluntary health insurance", most is employment based, and includes many different types of health plan providers such as commercial insurance companies (e.g. United Health Group, Cigna, Well Point, and Aetna), Blue Cross/Blue Shield, and managed care organizations · Group insurance can be obtained through an employer, a union, or a professional organization, often advertised as a program, risk is spread out among the many insured, provides lower costs · In a self-insured plan, employer act as its own insurer instead of obtaining insurance through an insurance company, employers budget a certain amount to pay medical claims incurred by their employees, used by mainly large businesses but small business can also self-insure, can protect themselves against any potential risk of high losses by purchasing reinsurance or stop-loss coverage from a private insurance company, gives employers a greater degree of control in which costs are contained, and employers are exempt from premium taxes, certain mandatory benefits required for plans often in certain states, state insurance regulations such as reserve requirements and consumer protection requirements · Individually purchased private health insurance is available as well, risk indicated by everyone's health status and demographics, and high-risk individuals were often unable to obtain privately purchased health insurance until ACA

What are Medicare trust funds? Discuss the current state and the future challenges faced by the Medicare trust funds. Which main factors pose these challenges?

· Medicare has established two main trust funds: the HI trust fund provides the money pool for Part A services and the SMI trust fund provides the money pool for Parts B and D. Each trust fund accounts for its own incomes and expenditures. Taxes, premiums, and other revenues are credited to the respective trust funds and benefit payments and administrative costs are the only purposes for which disbursements from the funds can be made. HI trust fund financed until 2028 and SMI trust fund financed through 2026 · 3 factors raising concerns about the future solvency of Medicare: the cost of delivering health care continues to grow at a rate faster than the rate of inflation in the general economy, an aging population will consume a greater quantity of health care services, and the workforce has been shrinking, and wage increases to support payroll tax revenues have been smaller than the rise in medical inflation

Discuss the concept of value-based purchasing as required by the ACA

· Medicare uses two different tracks for value-based reimbursement: Merit-Based Incentive Payment (ties payment bonuses or penalties to quality measures, resource use compared to similar care episodes and clinical conditions, care coordination and shared decision making, and use of electronic health records and Advanced Alternative Payment Methods (paid through the Medicaid Shared Savings Program as a result of participating in accountable care organizations and patient-centered medical homes) · Medicare has enforced a quality reporting program for certain types of facilities

Distinguish between national health expenditures and personal health expenditures

· National health expenditures=an aggregate of the amount a nation spends for all health services and supplies, public health services, health-related research, administrative costs, and investment in structures and equipment during a calendar year · Personal health expenditures=comprise the total spending for services and goods related directly to patient care

What is Medicare Part B? Discuss the financing and cost-sharing features of Medicare Part B. Which main benefits are covered under Part B? Which services are not covered?

· Part B= the supplementary medical insurance portion of Medicare, a voluntary program financed partly by required premium contributions, most are income-based, almost all entitled to Part A can enroll in Part B because they cannot get similar coverage at the same price from private insurers, ACA provided for annual physical exam for all Part B enrollees without any cost sharing Beneficiaries bear approximately 25% of the cost of premiums, Deductible included along with 80/20 split of coinsurance Main Benefits covered under Part B: Physician services, emergency department services, outpatient surgery, diagnostic tests and laboratory services, outpatient physical therapy, occupational therapy, and speech therapy, outpatient mental health services, limited home health care under certain conditions, ambulance, renal dialysis, artificial limbs and braces, blood transfusions and blood components, organ transplants, medical equipment and supplies, rural health clinic services, and annual physical exam: wellness exam and preventive services as medically needed: alcohol misuse screening and counseling, bone mass measurement, mammography, cardiovascular screening, Pap smears, colorectal cancer screening, depression screening, diabetes screening, glaucoma tests, HIV screening, nutritional counseling for diabetes and renal disease, obesity screening and counseling, prostate cancer screening, sexually transmitted infections screening, shots (flu, pneumococcal, hepatitis B), and tobacco use cessation counseling Noncovered services under Part B: Dental services, hearing aids, eyeglasses (except after cataract surgery), and services not related to treatment or injury

Briefly describe the Medicare Advantage program

· Part C= provides additional choices of health plans, beneficiaries receive both Part A and Part B services through a MCO and prescription drug under Part D are also included if offered by MCO, reduces the need to purchase Medigap coverage, has lower out-of- pocket costs

Briefly explain the prescription drug program under Medicare Part D

· Part D= available to anyone who has coverage under Part A or Part B, requires payment of a monthly premium to Medicare, coverage is offered through 2 types of private plans approved by Medicare: Stand-alone prescription drug plans (offer only drug coverage for those who want to stay in the original Medicare fee-for-service program) and Medicare Advantage Prescription Drug plans (MA-PDs) available to those who are enrolled in Part C if the MCO provides prescription drug coverage, also requires payment of a deductible following which a basic level of coverage becomes available, a coverage gap requires the beneficiary to pay the full cost of drug until a defined level of spending is reached, special provisions in the program are designed to help low-income enrollees by keeping their out-of-pocket costs to a minimum, all Part D drugs must be covered under a manufacturer discount agreement with the CMS, and beneficiaries receive discounts on drugs while in the coverage gap Initial coverage=Medicare pays 75% of the cost of drugs until the combined total payments by the plan and the beneficiary reach $3,700 Coverage gap=Beneficiary pays 40% of the cost for brand name drugs, and 51% of the cost for generic drugs and ends when the beneficiary has spent $4,950 out of pocket for brand-name drugs, the manufacturer's discount also counts toward out-of-pocket spending Catastrophic level= Beneficiary pays a small coinsurance (about 5%) or copayment Also there is an extra help program for those who have low incomes and savings, most serves those who receive Medicaid or Supplementary Security Income, and for those who qualify the out-of-pocket costs are minimal

Describe the major types of health service professionals (physicians, nurses, dentists, pharmacists, physician assistants, nurse practitioners, certified nurse midwives) including their roles, training, practice requirements, and practice settings

· Pharmacists dispense medicines prescribed by physicians, dentists, and podiatrists, and to provide consultation on the proper selection and use of medicines. All states require a license to practice pharmacy. The licensure requirements traditionally included graduation from an accredited pharmacy program that awards a Bachelor of Pharmacy or Doctor of Pharmacy (PharmD) degree, successful completion of a state board examination, and practical experience or completion of a supervised internship. Most pharmacists hold salaried positions and work in community pharmacies that are independently owned or are a part of a national drug store, discount store, or department store chain. Pharmacists are also employed by hospitals, MCOs, home health agencies, clinics, government health services organizations, and pharmaceutical manufacturers. In recent decades, the role of pharmacists has shifted from preparing and dispensing medicine to include educating on drug products and serving as experts on specific drugs, drug interactions, and generic drug substitution.

Describe the major types of health service professionals (physicians, nurses, dentists, pharmacists, physician assistants, nurse practitioners, certified nurse midwives) including their roles, training, practice requirements, and practice settings

· Physicians play a central role in the delivery of health services by evaluating patients and health conditions, diagnosing abnormalities, and prescribing treatments. All Physicians are required to be licensed to practice in each state. The licensure requirements include graduation form an accredited medical school that awards a Doctor of Medicine (MD) or Doctor of Osteopathic Medicine (DO) degree; successful completion of a licensing examination governed by either the National Board of Medical Examiners or the National Board of Osteopathic Medical Examiners, and completion of a supervised internship/residency program. Residency= graduate medical education in a specialty that takes the form of paid on-the-job training and usually occurs in a hospital. Before entering a residency, which may last 2 to 6 years, most DOs serve a 12-month rotating internship after graduation. Physicians practice in a variety of settings and arrangements. Some work in hospitals as medical residents or staff physicians, others work in the public sector such as federal government agencies, public health departments, community and migrant health centers, schools, and prisons. Most physicians are office-based practitioners and most physician contacts occur in physician offices. An increasing number of physicians are partners or salaried employees, working in both hospitals and various outpatient settings, such as group practices, freestanding ambulatory care clinics, and diagnostic imaging centers

Discuss how the concepts of premium, covered services, and cost sharing apply to health insurance

· Premium= the amount charged by an insurer to insure against specified risks, are determined by the actuarial assessment of risk that adjusts premiums to reflect health status, and 3 different methods used to determine them · Risk Rating= actuarial assessment of risk that adjusts premium to reflect health status · The 3 different methods used to determine premiums are experience rating, community rating, and adjusted community rating · Experience rating= based on a group's own medical claims experience, premiums differ under this method because different groups have different risks, and high-risk groups are charged higher premiums because of the expectation of incurring high utilization of medical care services · Community rating=spreads the risk among members of a larger population, premiums based on the utilization experience of the entire population covered by the same type of health insurance, same rate applies to everyone regardless of age, gender, occupation, or any other indicator of health risk · Adjusted community rating= also known as modified community rating, is a middle of the road approach that overcomes the main drawbacks of experience rating and pure community rating, under this method price differences take into account demographic factors such as age, gender, geography, and family composition, while ignoring other risk factors, required by the ACA · In addition to paying a share of the cost of premiums through payroll deductions, insured individuals pay a portion of the actual cost of medical services out of their own pockets which take the form of deductibles and copayments and are incurred only if and when medical services are used · Deductible= the amount the insured must first pay each year before any benefits are payable by the plan · Copayment=a flat amount the insured must pay each time health service are received · Health plans may also use coinsurance which is a set proportion of medical costs that the insured must pay out of pocket. Health plans generally have an annual maximum limit on out-of-pocket cost sharing and once that limit is reached the plan pays 100% of any additional expenses · The rationale of cost sharing is to control utilization of health care services · Benefits= services covered by an insurance plan and normally stated in the contract which medical services are covered and are not

What are the differences between the retrospective and prospective methods of reimbursement?

· Retrospective Reimbursement=the reimbursement methodology sets rates after evaluating the costs retrospectively · Prospective Reimbursement=uses certain established criteria to determine the amount of reimbursement in advance before services are delivered, minimizes the abuses from cost-plus approaches, enables providers to better predict future health care spending , provides strong incentives to health care organizations to reduce costs, used to reimburse inpatient hospital acute care services since 1983, was mandated in 1997 for hospital outpatient services, post-acute-care providers such as SNFs, home health agencies, and inpatient rehabilitation facilities

How does technology driven competition lead to greater levels of technology diffusion? How does technological diffusion, in turn, lead to greater competition? How does technology driven competition lead to duplication of services?

·-hospitals/outpatient centers compete to attract insured patients-patients seek "quality" care and are influenced by advertising state of the art technology (i.e. specialty hospitals) -medical arms race leads hospitals to offer new services lines that require advanced technology -to enhance perceptions of "quality", institutions/practices must recruit specialists who have been trained and require advanced technology to provide care -thus as a consequence, institutions/practices duplicate service

Discuss the government's role in the delivery and financing of health care, with specific reference to the dichotomy between public health and private medicine

• The federal government provides financing for MCR and MCD.• The federal government also sets standards for participating• Federal government delegates regulatory oversight to the state• Private hospitals and providers (physicians) deliver the care.• Scrutiny on quality of care is from the states through public health agencies for the delivery of individual care.• Reimbursement is for care, not health.

What are some of the social changes that led to the creation of specialized health centers for women?

-Recognition that women are the major users of health care. They seek health care more often than men do. Morbidity is greater among women than among men even after adjusting data for childbearing-related conditions -A change in philosophy in American culture toward women as the idea of gender equality becomes more popular -Recognition that the female majority in the US will continue to grow as the aging population includes more females

What are the conditions of eligibility for receiving home health services under Medicaid?

-The patient is homebound -The patient is under the care of a physician who will establish a home health plan - The patient needs physical therapy, occupational therapy, speech therapy, or intermittent skilled nursing care

What are some of the reasons why solo practitioners are joining group practices?

-Uncertainties created by rapid changes in the health care delivery system -Contracting by MCOs with consolidated organizations rather than solo entities -Competition from large health care delivery organizations -High cost of operating a solo practice -Complexity of billings and collections in a multiple-payer system -Increased external demands such as the necessity of having up-to-date IT systems Group practice and other organizational arrangements offer the benefits of a patient referral network, negotiated leverage with MCOs, sharing of overhead expenses, ease of obtaining coverage from colleagues for personal time off, and attractive starting salaries, with benefits and profit-sharing plans

Generally speaking, why is medical technology more readily available and used in the US than in other countries?

-anthro-cultural beliefs and values -medical specialization -financing and payment -technology-driven competition -expenditures on research and development -supply-side controls -government policy

Discuss the relationship between technological innovation and health care expenditures

-cost of adopting new technology includes: acquisition costs, labor costs, and facility costs -cost of new technology utilization: adoption of new technology leads to increased utilizations (episodic and chronic) and increased costs —although adopting of new tech may increase certain utilization costs in one area, they may decrease costs in other areas* —i.e. outpatient treatments reduce hospital admissions; robotics may replace human personnel (and improve accuracy), reducing labor costs The most important question to ask is whether the additional cost is associated with additional benefit (i.e. effect, quality, etc) - this is the value question!!

Why is it important to achieve a balance between clinical efficacy and economic worth (cost effectiveness) of medical treatments?

-economic resources are finite -in the absence of balancing costs and effects, the most efficient treatment alternatives will not be identified -will require a change in American mindset

What are some of the ethical issues surrounding the development and use of medical technology?

-how to provide the latest and best in health care within a limited resource parameters -conflicts of interest by providers who have stake in a tech and are responsible for its evaluation -insurers deciding when to cover novel treatments -conduct of clinical research

Which factors have been responsible for the low diffusion and low use of telemedicine?

-unclear or unidentified need -uncertain reimbursement policies -legal/licensing issues (reciprocity) -lack of universal access to technology -concerns about confidentiality -liability

Quality of Life (QOL)

1) Factors considered important by patients, such as environmental comfort, security, interpersonal relations, personal preferences, and autonomy in making decisions when institutionalized and 2) Overall satisfaction with life during and following a person's encounter with the health care delivery system

Allied Health

A broad category that includes services and professionals in many health-related technical areas. Allied health professionals include technicians, assistants, therapists, and technologists

Hospice

A cluster of special services for the dying, which blends medical, spiritual, legal, financial, and family-support services. The venue can vary from a specialized facility to a nursing home to the patient's own home

Adult Day Care

A community-based, long term care service that provides a wide range of health, social, and recreational services to elderly adults who require supervision and care while members of the family or other informal caregivers are away at work

Prepaid Plan

A contractual arrangement under which a provider must provide all needed services to a group of members (or enrollees) in exchange for a fixed monthly fee paid in advance to the provider on a per-member basis

Smart Card

A credit card-like device with an embedded computer chip and memory to hold personal medical information that can be accessed and updated at a hospital or physician's office

Claim

A demand for payment of covered medical expenses sent to an insurance company

Medically Underserved

A designation determined by the federal government that indicates a dearth of primary care providers and delivery settings, as well as poor health indicators of the populace; The majority of this population group are Medicaid recipients

Social Justice

A distribution principle, according to which health care is most equitably distributed by a government-run national health care program

Market Justice

A distributional principle according to which health care is most equitably distributed through the market forces of supply and demand, rather than government interventions

Copayment

A flat amount the insured must pay each time health services are received

Walk-In Clinic

A freestanding, ambulatory clinic in which patients are seen without appointments on a first come, first served basis

According to the Institute of Medicine, what are the four main components of a fully developed electronic health record (EHR) system?

A fully developed EHR system has 4 components: collection and storage of health information on individual patients over time, immediate electronic access to person- and population-level information by authorized users, availability of knowledge and decision support that enhances the quality, safety, and efficiency of patient care, and support of efficient processes for health care delivery

Free Clinic

A general ambulatory care center serving primarily the poor and the homeless who may live next to affluent neighborhoods; Free clinics are staffed predominantly by trained volunteers, and care is given free or at a nominal charge

Advanced Practice Nurse

A general name for nurses who have education and clinical experience beyond that required of an RN. APNs include four areas of specialization in nursing: clinical nurse specialists (CNSs), certified registered nurse anesthetists (CRNAs), nurse practitioners (NPs), and certified nurse midwives (CNMs).

Means-Tested Program

A government-run health insurance program in which eligibility depends on people's financial resources

Quad-Function Model

A health care delivery system incorporates four functional components: financing, insurance, delivery, and payment hence termed a quad-function model

Entitlement

A health care program to which certain people are entitled by right; For example, almost everyone at 65 years of age is entitled to Medicare coverage because of contributions made through laws

Self-Insured Plan

A health plan in which a large company acts as its own insurer by collecting premiums and paying claims. Such businesses most often purchase reinsurance against unusually large claims

Fee Schedule

A list of fees charged for various health care services

Gross Domestic Product (GDP)

A measure of all the goods and services produced by a nation in a given year

Insurance

A mechanism for protection against risk

Osteopathic Medicine

A medical philosophy based on the holistic approach to treatment. It uses the traditional methods of medical practice, which include pharmaceuticals, laboratory tests, x-ray diagnostics, and surgery and supplements them by advocating treatment that involves correction of the position of the joints or tissues and emphasizes diet and environment as factors that might destroy natural resistance

Prospective Reimbursement

A method of payment in which certain preestablished criteria are used to determine the appropriateness of utilization before the care is actually delivered

Community Health Assessment

A method used for conducting broad assessments of populations at a local or state level

Pharmaceutical Care

A mode of pharmacy practice in which the pharmacist takes an active role on behalf of patients, which includes giving information on drugs and advice on their potential misuse and assisting prescribers in appropriate drug choices. In so doing, the pharmacist assumes direct responsibility, in collaboration with other health care professionals and with patients, to achieve the desired therapeutic outcomes

Medicare Physician Fee Schedule (MPFS)

A national price list for physician services established by Medicare

Nanomedicine

A new area, still in its infancy, which involves the application of nanotechnology for medical use. Nanotechnology is a cutting-edge advancement within science and engineering. It is not a single field but an intense collaboration between disciplines to manipulate materials on the atomic and molecular level (one nanometer is one-billionth of a meter)

Retrospective Reimbursement

A payment scheme in which reimbursement rates are based on costs actually incurred

Cost Plus Reimbursement

A payment scheme in which reimbursement to a provider is based on cost plus a factor to cover the value of capital

Instrumental Activities of Daily Living (IADLs)

A person's ability to perform household and social tasks, such as home maintenance, cooking, shopping, and managing money

Adverse Selection

A phenomenon in which individuals who are likely to use more health care services than other persons due to their poor health enroll in health insurance plans in greater numbers compared to people who are healthy

Churning

A phenomenon in which people gain and lose health insurance periodically

Holistic Medicine

A philosophy of health care that emphasizes the well-being of every aspect of a person, including the physical, mental, social, and spiritual aspects of health

Allopathic Medicine

A philosophy of medicine that views medical treatment as active intervention to counteract the effects of disease through medical and surgical procedures that produce effects opposite those of the disease

Almshouse

A poorhouse; An unspecialized institution existing during the 18th and mid-19th centuries that mainly served general welfare functions, essentially providing shelter to the homeless, the insane, the elderly, orphans, and the sick who had no family to care for them

Medicare

A program for the elderly and certain disabled individuals which is administrated by the federal government to provide health insurance

Medicaid

A program for the poor and needy which is jointly administrated by the federal government and state governments to provide health insurance

Means Test

A program in which eligibility depends on income

Capitation

A reimbursement mechanism under which the provider is paid a set monthly fee per enrollee (sometimes referred to as per member per month or PMPM rate), regardless of whether or not an enrollee sees the provider and regardless of how often an enrollee sees the provider

Clinical Trial

A research study, generally based on random assignments, designed to study the effectiveness of a new drug, device, or treatment

Coinsurance

A set proportion of the medical costs that the insured must pay out of pocket when health care services are received

Community Rating

A system in which all members of a community are charged the same insurance rate

Managed Care

A system of health care delivery that seeks to achieve efficiency by integrating the four functions of health care delivery, employs mechanisms to control (manage) utilization of medical services, and determines the price of services and consequently how much the providers paid

What is managed care?

A system of health care delivery that seeks to achieve efficiency by integrating the four functions of health care delivery, employs mechanisms to control(manage) utilization of medical services, and determines the price of services and consequently, how much the providers are paid

Underwriting

A systematic technique used by an insurer for evaluating, selecting (or rejecting), classifying, and rating risks

Play-or-Pay

A type of employer mandate in which employers must choose to provide health insurance to employees ("play") or pay a penalty

Pesthouse

A type of facility operated by local governments during the 18th and mid-19th centuries to quarantine people who had contracted a contagious disease such as cholera, smallpox, or typhoid. The primary function of a pesthouse was to protect the community from the spread of contagious disease; Medical care was a secondary consideration

Public Health

A wide variety of activities undertaken by state and local governments to ensure conditions that promote optimum health for society as a whole

Third Party

Act as intermediates between the financing and delivery functions

Life Expectancy

Actuarial determination of how long, on average, a person of a given age is likely to live

Emergent Conditions

Acute conditions that require immediate medical attention

The Blum model points to four key determinants of health. Discuss their implications for health care delivery

All of these factors must be considered simultaneously when addressing the health status of an individual or a population. The most important is environment followed by lifestyle and heredity and medical car has the least impact on health and well-being in this model. Environmental factors encompass the physical, socioeconomic, sociopolitical, and sociocultural dimensions. The positive relationship between socioeconomic status and health is explained by people who have better education also have higher incomes thus can receive better health care. The joint relationship of income inequality and availability of primary care has also been found to be significantly associated with individuals self-rated health status. Lifestyle factors can affect how much can be done health care wise. Advances in gene therapy hold promise of treating a variety of inherited or acquired diseases. The access to primary care correlates with reduced mortality, increased life expectancy, and improved birth outcomes.

Population at Risk (PAR)

All the people in the same community or population group who are susceptible to acquiring a disease or a negative health condition

In general, who are allied health professionals? What role do they play in the delivery of health services?

Allied Health Professional=someone who has received a certificate, associate, bachelor's, or master's degree, doctoral-level preparation, or post-baccalaureate training in a science related to health care and has responsibility for the delivery of health or related services. These services may include those associated with the identification, evaluation, and prevention of diseases and disorders, dietary and nutritional services, rehabilitation, and health system management.

Alternative Medicine

Also called alternative and complementary medicine; Nontraditional remedies, such as acupuncture, homeopathy, naturopathy, biofeedback, yoga exercises, chiropractic, and herbal therapy

Adjusted Community Rating

Also called modified community rating; A method of determining health insurance premiums that takes into account demographic factors such as age, gender, geography, and family composition, while ignoring other risk factors

Supply-Side Rationing

Also called planned rationing; Government efforts to limit the availability of health care services, particularly expensive technology

Planned Rationing

Also called supply-side rationing; Government efforts to limit the availability of health care services, particularly expensive technology

Cross-Subsidization

Also known as cost shifting; In general, shifting of costs from one entity to another as a way of making up losses in one area by charging more in other areas. For example, when care is provided to the uninsured, the provider compensates for the costs for those services by charging more to the uninsured

Cost-Efficiency

Also known as cost-effectiveness, a state in which the benefit received from a service is greater than the cost incurred to provide that service

Cost Shifting

Also known as cross-subsidization; In general, shifting of costs from one entity to another as a way of making up losses in one area by charging more in other areas. For example, when care is provided to the uninsured, the provider compensates for the costs for those services by charging more to the uninsured

Efficacy

Also known as effectiveness; the health benefits of a medical intervention

Effectiveness

Also known as efficacy; the health benefits of a medical intervention

Licensed Practical Nurses (LPN)

Also known as licensed vocational nurses (LVNs) in some states; Nurses who have completed a state-approved program in practical nursing and a national written examination. LPNs often work under the supervision of registered nurses to provide patient care

Chronic Condition

Also referred to as chronic disease; A medical condition that persists over time (3 months or longer);Chronic diseases may lead to a permanent medical condition that is nonreversible and/or leaves residual disability

Ambulatory Care

Also referred to as outpatient services; Ambulatory care includes (1) care rendered to patients who come to physicians' offices, outpatient departments of hospitals, and health centers to receive care; (2) outpatient services intended to serve the surrounding community (community medicine); and (3) certain services that are transported to the patient

Why did medicine have a domestic, rather than professional, character in the preindustrial era? How did urbanization change that?

American attitudes about medical treatment placed strong emphasis on natural history and conservative common sense. Urbanization created increased reliance on the specialized skills of paid professionals in several ways by making them more accessible and more affordable. People were further from families (having moved from rural to urban settings), women began working outside the home. Opportunity costs to consult with doctors decreased. Travel was faster, phones could be used.

Why did the professionalization of medicine start later in the US than in some Western European nations?

American medicine lagged behind the advances in medical science, experimental research, and medical education because Americans had a tendency to neglect research in basic sciences and to place more emphasis on applied science. The medical profession was seen as a trade in early medicine. Medical practice was in disarray early on - it was informal, unregulated and had no existing standards for care• Medical procedures were primitive (e.g., bloodletting) and there was little science to back things up• An institutional core was missing - there were very few hospitals before 1800• Demand was unstable. Very few physicians were rural; most were in big cities and opportunity cost to get to physicians was high (travel, missed work, etc.)• Medical education was substandard. It was typical to have an apprenticeship rather than a formal education.

What are the two main objectives of a health care delivery system?

An acceptable health care delivery system should have two primary objectives: 1) Enable all citizens to obtain needed health care services and 2)Ensure that services are cost-effective and meet certain established standards of quality

Third Party Administrator (TPA)

An administrative organization, other than the employee benefit plan or health care provider, that collects premiums, pays claims, and/or provides administrative services

Case Mix

An aggregate of the severity of conditions requiring medical intervention; Case-mix categories are mutually exclusive and differentiate patients according to the extent of resource use

Maldistribution

An imbalance of the distribution of health professionals such as physicians, needed to maintain the health status of given population at an optimum level. Geographic maldistribution refers to the surplus in some regions but shortage in other regions of needed health professionals. Specialty maldistribution refers to the surplus in some specialties but shortage in others

Health Information Organization (HIO)

An independent organization that brings together health care stakeholders within a defined geographic area and governs electronic information exchange among these stakeholders with the objective of improving the delivery of health care in the community

Insurer

An insurance agency or managed care organization that offers insurance

Group Insurance

An insurance policy obtained through an entity, such as an employer, a union, or a professional organization, under the assumption that a substantial number of people in the group will participate in purchasing insurance through that entity

Case Management

An organized approach to evaluating and coordinating care, particularly for patients who have complex, potentially costly problems that require a variety of services from multiple providers over an extended period

Epidemic

An outbreak of an infectious disease that spreads rapidly and affects many individuals within a population

Cost-Effectiveness Analysis

Analysis that goes a step beyond the determination of efficacy(i.e. the benefit derived from the use of technology) by evaluating the additional (marginal) benefits to be derived in relation to the additional (marginal) costs to be incurred

Cost-Utility Analysis

Analysis that includes the use of quality-adjusted life years

Outpatient Services

Any health care services that are not provided on the basis of an overnight stay in which room and board are incurred

Socialized Medicine

Any large-scale government-sponsored expansion of health insurance or intrusion in the private practice of medicine

Health Technology Assessment (HTA)

Any process of examining and reporting properties of a medical technology used in health care, such as safety, effectiveness, feasibility, and indications for use, cost, and cost effectiveness, as well as social, economic, and ethical consequences, whether intended or unintended

E-Therapy

Any type of professional therapeutic interaction that makes use of the Internet to connect qualified mental health professionals and their clients

Beneficiary

Anyone covered under a particular health insurance plan

Why is there an imbalance between primary care and specialty care in the US?

Approximately 47.7% of physicians work in primary care while the remaining 52.3% are specialists. Most residents report subspecialty career plans for graduation. One in six general internists leave their practice midcareer either due to dissatisfaction or moving into a subspecialty of internal medicine. Medical students are more attracted to go into subspecialities because their training is organzed around medical technologies. Growth of new medical technology is one major driving force behind the increasing number of specialties along with the higher incomes of specialists relative to PCPs contributing to an oversupply of specialists

Discuss the global trends in primary care

Around the world, there is little consistency in how primary care services are accessed and how physicians get paid. In the UK, Netherlands, and New Zealand, patients register with a primary care doctor. In Australia, the Netherlands, New Zealand, Norway, Sweden, Iceland, Italy, Denmark, and the UK, patients go through primary care for referrals to specialists and are often required to register with primary care practices (except in Australia). Canada, France, and Germany use financial incentives to encourage registration with primary care practices and coordinated referrals. The German "sickness funds" (insurance plans) offer an enrollment option. The UK offers the most comprehensive coverage with little or no patient cost sharing. Canada covers physician visits in full but medication coverage varies by province. Australia, New Zealand, and Germany include varying degrees of cost sharing. Other countries may use moderate cost sharing. In Australia, Canada, France, Germany, Switzerland, and US, payers typically use fee-for-service payments and employ performance incentives. Conversely, the Netherlands, New Zealand, Norway, Denmark, and UK use a combination of capitation, fee for service, and incentives. Approximately 59% of doctors in France, the Netherlands, New Zealand, and Switzerland report that their patients can get same- or next-day appointments when sick compared to only 22% in Canada. In Australia, Canada, France, Germany, New Zealand, and Norway, more than 60% of providers report long waits to see specialists. In the US, physicians complain that they spend a significant amount of time with insurance-related issues which limits access to care for their patients. Primary care is mostly privatized in all the previous countries mentioned with the exception of Iceland (mostly public) and Sweden (mixed). Physicians in Australia, Canada, Norway, the UK, and US are more likely to work in group practices of five or more doctors. Physicians in the Netherlands, France, Germany, and Switzerland are more likely to work in smaller practices. A survey reported that doctors in 10 countries struggle to coordinate care and communicate with other health and social services providers, and questioned their preparedness to care for patients with challenging issues. There has been conceptual convergence in recent years on the need to redesign primary care to meet the health care needs of aging populations and address the increased prevalence of chronic disease around the world (Schoen et al., 2012)

Describe Australia's Health Care System

Australia uses a national program called Medicare which is financed by income taxes and an income-based Medicare levy. 49% of Australia also has private health insurance to cover gaps in public coverage such as dental services and care received in private hospitals. This is voluntary but is recommended. If not purchased tax penalties are applied. Public hospital spending is funded by the government while private hospital offer better choices. Costs incurred by patients receiving private medical services whether in or out of the hospital are reimbursed in whole or in part by Medicare. Private patients are free to choose and change their doctors. The medical profession in Australia is composed mainly of private practitioners who provide care predominantly on a fee-for-service basis

Demand-Side Rationing

Barriers to obtaining health care faced by individuals who do not have sufficient income to pay for services or purchase health insurance

Biologics

Biological products such as vaccines, blood and blood components, allergenics, somatic cells, gene therapy, tissues, and recombinant therapeutic proteins

Describe Canada's Health Care System

Canada uses national health insurance system referred to as Medicare which consists of 13 provincial and territorial health insurance plans sharing basic standards of coverage. The bulk of financing for Medicare comes from general provincial tax revenues. The federal government provides a fixed amount that is independent of actual expenditures. Many employers also offer private insurance that gives their employees supplemental coverage. Patients are free to select their providers. The majority of Canadian hospitals are operated as private nonprofit entities run by community board of trustees, voluntary organizations, or municipalities and most physicians are in private practices. Most provinces use global budgets and physicians are paid fee-for-service rates which are negotiated between each provincial government and medical association

Specialty Care

Care that tends to be limited to illness episodes, the organ system, or the disease process involved. Specialty care, if needed, generally follows primary care

Orphan Drugs

Certain new drug therapies for conditions that affect fewer than 200,000 people in the United States

Subacute Condition

Clinically complex services that are beyond traditional skilled nursing care

Medigap

Commercial health insurance policies purchased by individuals covered by Medicare to insure the expenses not covered by Medicare

Describe the main public and voluntary outpatient clinics and the main problems they face

Community Health Centers address health care needs in medically undeserved regions of the US and provide services to anyone seeking care regardless of insurance status or ability to pay. Hence, CHCs are a primary care safety net for the nation's poor and uninsured in both inner-city and rural areas. CHCs are private nonprofit organization but receives federal grant funding through the Section 330 of Public Health Service Act. These centers also heavily depend on funding through the Medicaid program while private-pay patients are charged on sliding-fee scales determined by the patient's income. CHCs tailor their services to family-oriented primary and preventative health care and dental services. CHCs provide accessible, cost-effective, and quality care and play an important role in reducing health disparities among vulnerable populations. Free Clinics are general ambulatory care centers serving primarily the poor, the homeless, and the uninsured. Free clinics have 3 main characteristics: 1) Services are provided at no charge or at a very nominal charge, 2) The clinic is not directly supported or operated by a government agency or health department, and 3) Services are delivered mainly by trained volunteer staff. Free clinics focus on the delivery of primary care but other services offered vary depending on the staff training. The number of free clinics has continued to grow nationally. Federal funding is used to operate migrant health centers that serve transient farm workers in agricultural communities and rural health centers that serve populations in isolated underserved rural areas. The Community Mental Health Center program was established to provide ambulatory mental health services in underserved areas. These various types of clinics generally face serious problems because of inadequate funding and a shortage of primary care providers. Other challenges include the difficulty of recruiting and retaining physicians and other qualified health professionals and the inability to diversify patient mix, particularly regarding patients with private insurance or those who have the ability to pay for services

What is community-oriented primary care? Explain.

Community-oriented primary care (COPC) incorporates the elements of good primary care delivery and adds to this a population-based approach to identifying and addressing community health problems. It is based on the biopsychosocial paradigm, which is broader in scope than the more restricted medical model of healthcare delivery. The former emphasizes the health of the population as well as the individual. The main challenges behind COPC are bringing together individual health needs in the larger context of community health needs, a lack of consensus on what a community is or should be, workforce shortages, and financial incentives. COPC would require a major transformation of the current system and face the same implementation problem as medical homes

What is complementary and alternative medicine? What role does it play in the delivery of health care?

Complementary and Alternative Medicine (CAM)=nonconventional therapies or natural medicine. CAM refers to the broad domain of all health care resources other than those intrinsic to biomedicine and covers a heterogenous spectrum of ancient to new approaches that purport to prevent or treat disease. Complementary treatments are used together with conventional medicine whereas alternative medicine is used instead of conventional medicine. CAM therapies include a wide range of treatments such as homeopathy, herbal formulas, use of other natural products as preventative and treatment agents, acupuncture, meditation, yoga exercises, biofeedback, and spiritual guidance or prayer. Chiropractic is also largely regarded as a CAM treatment. No particular settings of health care delivery are involved in CAM treatments. Most therapies are self-administered or at least require active patient participation. CAM is used for a wide array of diseases and conditions and is popular in Europe, Canada, and other industrialized countries. A doctor of naturopathic medicine (ND) degree or Diplomate of the Homeopathic Academy of Naturopathic Physicians (DHANP) required to practice this. Natural medicine-based private clinics are emerging across the US.

Decision Support Systems

Computer-based information and analytical tools to support managerial decision making in health care organizations

Organized Medicine

Concerted activities of physicians, mainly to protect their own interests, through such associations as the American Medical Association (AMA).

Nonurgent Conditions

Conditions that do not require the resources of an emergency service, and in which the disorder is nonacute or minor in severity

Urgent Conditions

Conditions that require medical attention within a few hours; A longer delay presents possible danger to the patient; This kind of disorder is acute but not necessarily severe

System

Consists of a set of interrelated and interdependent, logically coordinated components designed to achieve common goals

Primary Care

Continual basic and routine care

Why does cost containment remain an elusive goal in US health services delivery?

Cost containment is difficult to attain in US health service delivery because of the large number of entities that are loosely networked without a singular overseeing body

Administrative Costs

Costs associated with billing, collections, bad debts, and maintaining medical records

Mortality

Death

Medical Model

Delivery of health care that places its primary emphasis on the treatment of disease and relief of symptoms instead of prevention of disease and promotion of optimum health

Describe the major types of health service professionals (physicians, nurses, dentists, pharmacists, physician assistants, nurse practitioners, certified nurse midwives) including their roles, training, practice requirements, and practice settings

Dentists diagnose and treat dental problems related to the teeth, gums, and tissues of the mouth. All dentists must be licensed to practice. The licensure requirement include graduation from an accredited dental school that awards a Doctor of Dental Surgery (DDS) or Doctor of Dental Medicine (DMD) degree and successful completion of both written and practical examinations. Some states require dentists to obtain a specialty license before practicing as a specialist in that state. Nine specialty areas in Dentistry: Orthodontics (straightening teeth), oral and maxillofacial surgery (operating · on the mouth and jaws), oral and maxillofacial radiology (producing and interpreting images of the mouth and jaws), pediatric dentistry (dental care for children), periodontics (treating the gums), prosthodontics (making artificial teeth or dentures), endodontics (root canal therapy), public health dentistry (community dental health), and oral pathology (diseases of the mouth). The growth of dental specialties is influenced by technological advances, including implant dentistry, laser-guided surgery, orthognathic surgery (surgery performed on the bones of the jaw) for the restoration of facial form and function, new metal combinations for use in prosthetic devices, new bone graft materials in "tissue guided regeneration" techniques, and new materials and instruments. Many dentists are involved in the prevention of dental decay and gum disease. Dental· prevention includes regular cleaning of patients' teeth and educating patients on proper dental hygiene. Dentists also spot symptoms that require treatment by a physician. Dentists employ dental hygienists and assistants to perform many of the preventive and routine care services. Most dentists practice in private offices solo or in a group. Dental offices operate as private businesses and dentists often perform business tasks such as staffing, financing, purchasing, leasing, and work scheduling. Some dentists are employed in clinics operated by private companies, retail stores, or franchised dental outlets. Group dental practices, which offer lower overhead and increased productivity, have grown slowly. The federal government also employs dentists mainly in hospitals and clinics of the Department of Veteran Affairs and the US Public Health Service

Global Budgets

Determines total health care expenditures on a national scale and allocate resources within budgetary limits thus as a consequence both availability of services and payments to providers are subject to each budgetary constraints

Describe Developing Countries Health Care System

Developing Countries (East Asia and the Pacific, Eastern Europe, Central Asia, Latin America, The Caribbean, The Middle East, North Africa, South Asia, and sub-Saharan Africa) are moving towards adopting universal health coverage to decrease the financial impoverishment due to health spending, improve health, and increase access to care. People with private financial means can find reasonably good health care. Majority of the population depend on limited governmental services that are often of questionable quality. Government financing for health services increases in countries with higher per-capita incomes. Trends in health reforms in developing countries include increasing enrollment in government-sponsored health insurance, expanded benefits packages, decreasing out of pocket expenditures, and increasing the governments share of health spending

Discuss the prospective payment system under DRGs

Diagnosis-Related Group Based Reimbursement= based on the principal diagnoses and case, rates adjusted for geographic differences, whether the institution is a teaching hospital and whether a hospital treats a disproportionate share of low-income patients, safety net hospitals, outliers in which long hospital stays or extremely expensive call for additional payments, hospital receives a predetermined fixed rate per discharge based on the patient's DRG classification and adjustment factors, give financial incentive for hospital to keep the length of stay as short as possible, Psych facilities paid daily rate based on national average daily costs for routine, ancillary, and capital costs and adjusted for certain factors, Long-Term Care Hospitals paid in 3 different ways: Daily payment rates set according to Medicare severity long-term care diagnosis-related groups for post-acute care and for ventilator-dependent patients but for others reimbursement is the cost of caring for the patient

Describe the major types of health service professionals (physicians, nurses, dentists, pharmacists, physician assistants, nurse practitioners, certified nurse midwives) including their roles, training, practice requirements, and practice settings

Doctoral Nursing Degrees include the Doctor of Nursing Practice (DNP), Doctor of Nursing Science (DNS), and Doctor of Philosophy in Nursing (PhD). The DNS and PhD degrees are research focused whereas the DNP emphasizes patient care and nursing practice. A doctoral degree is usually required to become a professor of nursing education or nurse researcher. Doctoral degrees are also preferred for nurse practitioners, clinical nurse specialists, nurse anesthetists, and nurse mid-wives. · All states require nurses to be licensed to practice. Nurses can be licensed in more than one state through examination or endorsement of a license issued by another state. The licensure requirements include graduation from an approved nursing program and successful completion of a national examination. Registered nurses (RNs) must complete an associate degree in nursing (ADN), a diploma program or baccalaureate degree in nursing (BSN). ADN programs take about to 3 years and offered by community and junior colleges. Diploma programs take 2 to 3 years and are still offered by a few hospitals. BSN programs take 4 to 5 years and are offered by colleges and universities. Licensed Practical Nurses (LPNs) must complete a state-approved program in practical nursing and a national written examination. Most practical nursing programs last about 1 year and include classroom study, as well as supervised · clinical practice. Nurse managers act as supervisors of other nurses and RNs supervise LPNs. · Advanced Practice Nurses (APN) have education and clinical experience beyond that required of an RN. APNs have four areas of specialization: clinical nurse specialists (CNSs), certified registered nurse anesthetists (CRNAs), nurse practitioners (NPs), and certified nurse-midwives (CNMs). APNs collaborate and consult with other health care professionals, educate patients and other nurses, collect data for clinical research projects, and participate in the development and implementation of total quality management programs, critical pathways, case management, and standards of care. Both CNSs and NPs work at hospitals, primary care, and other settings, CNSs can not prescribe drugs while NPs may prescribe drugs in most states, CRNAs are trained to manage anesthesia during surgery, and CNMs deliver babies and manage the care of mothers and healthy newborns before, during, and after delivery. The requirements for becoming an APN vary greatly from state to state. In general, the designation requires a graduate degree in nursing or certification in an advanced practice specialty area. Nurses work in a variety of settings. In addition, they work in home health care, hospice care, and long-term care settings. A few work as private-duty nurses in patients' homes. Nurses are often classified according to the settings in which they work.

What implications has the decline in hospital occupancy rates had for hospital management?

Due to declining occupancy rates, hospital executives have been forced to view ambulatory care as an essential portion of their overall healthcare business rather than a supplemental product line of an inpatient facility. Seeing their inpatient business erode, hospital administrators have realized that establishing a firm position in the ambulatory care market is critical to the continued survival of their organizations. To meet the growing demand for outpatient services, hospitals have expanded into services that previously were not considered a part of their core business

Telehealth

Educational, research, and administrative uses of telecommunications technology in healthcare, as well as clinical applications that involve nurses, psychologists, administrators, and other non physicians

Secondary Prevention

Efforts to detect disease in early stages so as to provide a more effective treatment-for example, screening

Global Health

Efforts to protect the entire global community against threats to people's health and to deliver cost-effective public health and clinical services to the world's population

What is the primary reason for employers to purchase insurance plans to provide health benefits to their employees?

Employees get group rates through their employer that are generally lower than what the rates would be if the employees were to purchase health insurance on their own

Premium Cost Sharing

Employers rarely pay 100% of the insurance premiums but instead most require their employees to pay a portion of the cost

Moral Hazard

Enrollees purchase health insurance but they may use more health care services than if they were to pay for these services on an out-of-pocket basis

Risk Factors

Environmental elements, personal habits, or living conditions that increase the likelihood of developing a particular disease or negative health condition in the future

Primary Care

Essential health care that constitute the first level of contact by a patient with the health delivery system and the first element of a continuing health care process

Primary Health Care

Essential health care that constitutes the first level of contact by a patient with the health delivery system and the first element of a continuing health care process (World Health Organization); The provision of integrated, accessible health care services by clinicians who are accountable for addressing a large majority of personal health care needs, developing a sustained partnership with patients, and practicing in the context of family and community (Institute of Medicine definition)

Cost-Benefit Analysis

Evaluation of benefits in relation to costs when both are expressed in dollar terms

Health Determinants

Factors that contribute to the general well-being of individuals and population

Which factors explain why the demand for services of a professional physician was inadequate in the preindustrial era? How did scientific medicine and technology change that?

Families and communities were accustomed to treating the sick using folk remedies passing from generation to generation in rural America. It was also common to consult published books and pamphlets that gave advice on home remedies. The market for physicians' services was limited by economic conditions. Many families could not afford to pay for medical services. Personal health services had to be purchased without the help of the government or private insurance. Science-based medicine created an increased demand for advanced services that were no longer available through family and neighbors

Part B

Federal government-subsidized voluntary insurance for physician services and outpatient services

To what extent do you think the objectives set forth in Healthy People initiatives can achieve the vision of an integrated approach to health care delivery in the United States?

For an integrated approach to become reality, the best American ingenuity must be applied in addressing health spending reductions and coordination of services among public health agencies, hospitals, and other healthcare providers so I do think the objectives set forth in Healthy People initiatives can achieve the vision of an integrated approach to health care delivery in the US

National Health Insurance (NHI)

Found in Canada; The government finances health care through general taxes but the actual care is delivered by private providers; In the context of the quad-function model, NHI requires a tighter consolidation of the financing, insurance, and payment functions coordinated by the government but delivery is characterized by detached private arrangements

Socialized Health Insurance (SHI)

Found in Germany; Government-mandated contributions from employers and employees finance health care, private providers deliver health care services, and private not-for-profit insurance companies called sickness funds are responsible for collecting the contributions and paying physicians and hospitals. The insurance and payment function are closely integrated in a SHI system and the financing function is better coordinated with the insurance and payment functions. Delivery is characterized by independent private arrangements but the government exercises overall control of the system

National Health System (NHS)

Found in the UK; The government manages the infrastructure for the delivery of medical care thus the government operates most of the country's medical institutions. Most health care providers are government employees or are tightly organized in a publicly managed infrastructure. In the context of the quad-function model, NHS requires a tighter consolidation of all four functions

Which type of illegal activities constitute health care fraud and abuse?

Fraud can take several forms and a single case can involve more than one fraud scheme. Examples of fraud include billing for services not provided, delivery of unnecessary services, providing compensation to others or receiving kickbacks for participating in a fraud scheme, and misrepresentation of services to receive higher reimbursement (GAO, 2016)

Surgicenters

Freestanding, ambulatory surgery centers that perform various types of surgical procedures on an outpatient basis

In general, discuss how technological, social, and economic factors created the need for health insurance

From a technological perspective, medicine offered new and better treatments. Because of its well-established healing values, medical care had become individually and socially desirable, which created a growing demand for medical services. From an economic perspective, people could predict neither their future needs for medical care nor the costs, both of which had been gradually increasing. In short, scientific and technological advances made health care more desirable but less affordable thus the need for some kind of insurance that spread the financial risks over a large number of people

Residency

Graduate medical education in a specialty that takes the form of paid on-the-job training, usually in a hospital

What are the main provisions of HIPAA with regard to the protection of personal health information? Which provisions were added to HIPAA under the HITECH Act?

HIPAA (1996): prohibited access to a patient's PHI for reasons other than health care delivery, operations, and reimbursement HITECH Act (2009): health info tech for economic and clinical health act-designated $19 billion in direct grants and financial incentives to adopt government certified EHRs by hospitals and physicians-penalties for non-implementation (tied to reimbursement)-established standards for EHRs and their use (aka meaningful use) Provisions that were added to HIPAA under the HITECH Act (4):1. extended HIPAA protections to include "business associates" of providers and restricts use of PHI in marketing2. patient authorization for research3. use of genetic info4. patients' right to receive electronic copies of PHI

Single-Payer System

Has only one primary payer the government

Distinguish between information technology (IT) and health informatics

Health Informatics= broadly defined as the application of information science to improve the efficiency, accuracy, and reliability of health care services. Health informatics requires the use of IT but goes beyond IT by emphasizing the improvement of health care delivery. Applications of informatics are also found in electronic health records and telemedicine

E-Health

Health care information and services offered over the internet by professionals and nonprofessionals alike

Physical Therapists (PTs)

Health care professionals who provide care for patients with movement dysfunction

Podiatrists

Health care professionals who treat patients with foot diseases or deformities

Dental Assistants

Health care professionals who usually work for dentists in the preparation, examination, and treatments of patients

Physician Assistants (PAs)

Health care professionals who work in a dependent relationship with a supervising physician to provide comprehensive medical care to patients. The major services provided by PAs include evaluation, monitoring, diagnostics, therapeutics, counseling, and referral

Dental Hygienists

Health care professionals who work under the supervision of dentists and provide preventative dental care, including cleaning teeth and educating patients on proper dental care

Occupational Therapists (OTs)

Health care professions who help people of all ages improve their ability to perform tasks in their daily living and working environments. OTs work with individuals who have conditions that are mentally, physically, developmentally, or emotionally disabling

Describe Germany's Health Care System

Health insurance has been mandatory for all citizens and permanent residents in Germany since 2009. Germany follows the SHI model and voluntary substitutive private health insurance is available. Its national system employs global budgets for the hospital sector and places annual limits on spending for physician services. Inpatient care is paid per admission based on diagnosis-related groups

Why is it that, despite public and private health insurance programs, some U.S. citizens are without health care coverage?

Health insurance is offered voluntarily by employers as a fringe benefit. Some employers, especially small businesses, cannot afford to provide health insurance to their employees. The unemployed generally cannot participate in an employer-sponsored program. Under current laws, those who have recently separated from their employment can generally obtain coverage at group rates for up to twelve months from their previous employer. They must, however, pay the entire premium, which many cannot afford. Even when premiums are subsidized by the employer, many low-wage workers cannot afford them. Participation in public programs, such as Medicare and Medicaid, requires meeting certain eligibility criteria. For instance, Medicare is only for the elderly and certain disabled individuals. Medicaid eligibility is determinedon the basis of assets and income, and generally only the very poor qualify.ACA is supposed to increase the number of people insured since it requires that all individuals must be covered by either public or private insurance and that insurance companies cover all applicants regardless of pre-existing conditions or sex.

High Deductible Health Plans (HDHPs)

Health plans that combine a savings option with a health insurance plan carrying a high deductible

Discuss the roles of efficacy, safety, and cost effectiveness in the context of health technology assessment

Health technology assessment (HTA): private industries◦ The evaluation of medical technology to determine: Efficacy, Safety, Cost-effectiveness ◦ The objective of HTA is to establish the appropriateness of medical technology for widespread use. ◦ Efficacy and safety are the basic starting points. Cost-effectiveness and cost-benefit go a step futher in evaluating the safety and efficacy in relation to the cost of using the technology. -efficacy: can the tech work under ideal conditions? (We make the conditions) -effectiveness: does the tech work under "real world" conditions? -safety: does the tech's primary benefits outweigh any negative consequences? -cost-effectiveness: integration of both costs and benefits - asks the question, are the additional effects worth the additional costs?

What are the major differences between Healthy People 2020 and the previous Healthy People 2020 initiatives?

Healthy People 2020 differs from previous Healthy People initiatives in that it includes multiple new topics areas to the objectives list, such as adolescent health, genomics, global health, health communication and health information technology, and social determinants of health. Healthy People 2020 has 42 topic areas with 13 new areas

Consumer-Directed Health Plans

High-deductible health plans that include a savings option to pay for routine health care expenses

What is the basic philosophy of home health care? Describe the services it provides

Home health is consistent with the philosophy of maintaining people in the least restrictive environment possible. Home health care services encompasses a wide range of services and supplies that a person receives at home under a plan of care established by a doctor. It can include skilled nursing and home health aide services, physical therapy, occupational therapy, speech-language pathology services, medical social services, DME (e.g. wheelchairs, hospital beds, oxygen, and walkers), medical supplies, and other services provided in the individual's home. Home health is delivered in the US by a combination of large and small home health providers both for profit and non profit. Home health providers often use remote monitoring including phone calls to health information technology to in-home therapeutic and diagnostic technologies. This enables home health providers to improve quality and reduce the cost of care delivered to patients

Explain the concept of hospice care and the types of services a hospice provides

Hospice care= cluster of comprehensive services for terminally ill persons with a medically determined life expectancy of 6 months or less. Hospice provide services that address the special needs of dying persons and their families. It is a method of care and services are taken to patients and their families wherever they are located. Hospice includes the following considerations: 1) meeting the patient's physical needs with an emphasis on pain management and comfort, 2) meeting the patient's and family's emotional and spiritual needs, 3) providing the support for the family members before and after the patient's death, and 4) focusing on maintaining quality of life rather than prolonging life. The two primary areas of emphasis in hospice care are palliation (pain and symptom management) and psychosocial and spiritual support. Counseling and spiritual help are made available to relieve anguish and help the patient deal with his or her death. Social services include help with arranging final affairs

All primary care is ambulatory, but not all ambulatory services represent primary care. Discuss

Hospital ED services are not intended to be primary in nature. Conversely, services other than primary care have now become an integral part of outpatient services. Thanks to the technological advances in medicine, many advanced treatments are now provided in ambulatory care settings. Examples include conditions requiring urgent treatment, outpatient surgery, renal dialysis, and chemotherapy

Discuss the main hospital-based outpatient services

Hospital-Based Outpatient Services: Clinical Services (Acquisition of group practices has enabled hospitals to increase their market share for outpatient clinical care, referrals for inpatient, surgical, and other specialized services have generated additional revenue for these hospitals, both public and private non profit hospitals provide services to patients who do not have access to private practitioner offices for routine care, and teaching hospitals operate various clinics offering highly specialized research-based services), Surgical Services ( Hospital-based ambulatory surgery centers provide same-day surgical care, patients are sent home after a few hours of recovery time following surgery, follow-up care generally continues in the physician's office), Emergency Services (have services available around the clock for patients who are acutely ill or injured particularly those with serious or life-threatening conditions requiring immediate attention, often overused by the uninsured and people on Medicaid for nonurgent or routine care that could be more appropriately addressed in a primary care setting, and can not turn down patients due to EMTALA), Home Health Services (provide mainly postacute care and rehabilitation therapies, hospitals have entered the home health business to keep discharged patients within the hospital system, and hospitals operate approximately 7.4% of all Medicare-certified home health agencies in the US), and Women's Health Services (rely on a variety of service delivery models that exist on a continuum that includes telephone information and referral, educational programs, health screening and diagnostics, comprehensive primary care for women, and mental health services, in addition to services in obstetrics, gynecology, and primary care, women's health centers offer mammography, ultrasound, osteoporosis screening, and other health screenings)

How did the emergence of general hospitals strengthen the professional sovereignty of physicians?

Hospitals symbolized the institutionalization of health care and became the central core around which the delivery of medical services was organized. For economic reasons, as hospitals expanded, their survival became increasingly dependent on physicians to keep the beds filled because the physicians decided where to hospitalize their patients. Therefore, hospitals had to make every effort to keep the physicians satisfied, which enhanced the physicians' professional dominance, even though they were not employees of the hospitals. In turn, physicians exerted enormous influence over hospital policy. Hospitals began conforming to both physician practice patterns and public expectations about medicine as a modern scientific enterprise

What role does an IT department play in a modern health care organization?

IT departments in health care organizations play a critical role in decisions to adopt new information technologies to improve health care delivery, increase organizational efficiency, and comply with various laws and regulations. Healthcare IT includes medical records systems to collect, transcribe, and store clinical data; radiology and clinical laboratory reporting systems; pharmacy data systems to monitor medication use and avoid errors, adverse reactions, and drug interactions; scheduling systems for patients, space such as surgery suites, and personnel; and financial systems for billing and collections, materials management, and many other aspects of organizational movement

Critique the gatekeeping role of primary care

If primary care physicians approve referrals to specialists too freely, they risk a reduction in their capitation rate or depletion of the fund reserved for specialty care and procedures. Also, specialists may complain about referrals for conditions that they think the primary care provider should have managed alone. Conversely, when primary care physicians infrequently open the gate to specialty care, patients may believe their physicians are denying them necessary care. Also, many patients visit specialists without reference

Iatrogenic Illness

Illness or injury caused by the process of medical care

Gatekeeping

Implies that patients do not visit specialists and are not admitted to a hospital without first being referred by their primary care practitioner

Free Market

In a free market system, patients (buyers) and providers (sellers) act independently with patients able to choose services from any provider. Providers do not collude to fix prices and prices are not fixed by an external agency. Rather, prices are governed by the free and unencumbered interaction of the forces of supply and demand.

Third-Party Payers

In a multipayer system, the payers for covered services, for example, insurance companies, managed care organizations, and the government. They are called third parties because they are neither the providers nor the recipients of medical services.

What is socialized health insurance (SHI)?

In a socialized health insurance system, health care is financed through government

Primary Prevention

In a strict epidemiologic sense, the prevention of disease- for example, health education, immunization, and environmental control measures

Describe how health care is rationed in the market justice and social justice systems

In market justice, healthcare is rationed using demand-side rationing in which prices and ability to pay ration the quantity and type of healthcare services that people consume so the uninsured and individuals who lack sufficient income to pay for private healthcare services face barriers to obtaining health care. To some extent, the uninsured may be able to overcome some barriers through charitable services. In social justice, healthcare is rationed using supply-side rationing in which the government decides how technology will be dispersed and who will be allowed access to certain types of costly high-tech services even though basic services may be available to all. Even when a covered individual has a medical need, depending on the nature of health services required, he or she may have to wait until services become available

Which conditions during the World War II period lent support to employer-based health insurance in the United States?

In order to control high inflation in the economy during the World War II period, Congress imposed wage freezes. In response, many employers started offering health insurance to their workers in lieu of wage increases.-In 1948, the US Supreme Court ruled that employee benefits, including health insurance, were a legitimate part of union-management negotiations. Health insurance then became a permanent part of employee benefits in the postwar era.-In 1954, Congress amended the Internal Revenue Code to make employer-paid health coverage nontaxable. In economic value, employer-paid health insurance was equivalent to getting additional salary without having to pay taxes on it, which provided an incentive to obtain health insurance as an employer-furnished benefit

On what basis were the elderly and the poor regarded as vulnerable groups for whom special government-sponsored programs needed to be created?

In principle, the poor were considered a special class who could be served through a government-sponsored program. The elderly-those 65 years of age and older- were another group who had started to receive increased attention in the 1950s. On their own, most of the poor and elderly could not afford the increasing costs of healthcare. Also, because the health status of these subpopulations was significantly worse than that of the general population and required a higher level of health care services. The elderly had a higher incidence and prevalence of a disease than compared to younger groups. It was also estimated that less than half of the elderly population was covered by private health insurance. Provision of charity care led to private payers being charged more for cost-shifting/cross subsidization

: Why is the US health care market considered to be "imperfect"?

In reality, US is not a free market, however, we have a quasi-market or imperfect market because the player is an MCO, Medicare or Medicaid and, rather than the patient. Prices are set by agencies external to the market; thus they are not freely governed by the forces of supply and demand

Health Care Reform

In the U.S. context, expansion of health insurance to cover the uninsured or those without private or public health insurance coverage

In the contrast of globalization in health services, which main economic activities are discussed in this chapter?

In the context of globalization in health services, the main economic activities that are discussed are the global exchange of information, production of goods and services in economically developed countries, and increased interdependence of mature and emerging world economies

Describe China's Health Care System

In urban China, they have a multipayer system. Government employees are covered under government insurance as a part of their benefits. Employees for public enterprises are largely covered through public enterprise insurance. Employees of foreign businesses or joint ventures are typically well insured through private insurance arrangements. The unemployed, self employed, and employees working for small enterprises are largely uninsured and can purchase individual/ family plans or pay for services out of pocket. In rural China, NCMS is widespread. Patients can now go to any hospital of their choice as long as they are insured or can pay out of pocket.

How did the organized medical profession manage to remain free of control by business firms, insurance companies, and hospitals until the latter part of the 20th century?

Individual physicians who took up practice in a corporate setting were pressured to abandon such practices. Independence from corporate control enhanced private entrepreneurship and put American physicians in an enviable strategic position in relation to hospitals and insurance companies. The American Medical Association help others advance professionally and financially.

Cases

Individuals who acquire a certain disease or condition

Nurse Practitioners (NPs)

Individuals who have completed a program of study leading to competence as RNs in an expanded role. NP specialties include pediatric, family, adult, psychiatric, and geriatric programs

Explain how contract practice and prepaid group practice were the prototypes of today's managed care plans

Industries operating contract practice plans either hired physicians on salary or contracted with independent physicians and hospitals at a flat capitated rate per worker per month. These features are found in certain managed care plans today. Prepaid group practice also provided comprehensive services for a capitated fee. Later, health maintenance organizations were modeled after prepaid health plans. Managed care organizations today attempt to combine the efficiencies of contract and group arrangements with the objective of delivering comprehensive healthcare services at predetermined costs

Electronic Health Records (EHRs)

Information technology applications that enable the processing of any electronically stored information pertaining to individual patients for the purpose of delivering health care services

Which "preparedness"-related measures have been taken to cope with potential natural and human made disasters since the tragic events of 9/11? Assess their effectiveness

Initiatives such as the Public Health Security and Bioterrorism Preparedness Response Act of 2002, Homeland Security Act of 2002, the Pandemic and All-Hazards Preparedness Act (PAHPA) of 2006 have been created along with Healthy People focused on improving the nation's ability to prevent, prepare for, respond to, and recover from a major health incident and The Health Alert Network was established in 2011 to facilitate communication, information, and distance learning related to health threats, including bioterrorism. Surge capacity is also important along with development in technology such as social media being used as a tool by the government, communities, and organizations for disaster preparedness. Despite the progress that has been made, disease preparedness efforts in the US remain fragmented and underfunded because of the differences in institutional and local structures, difficulty in developing clear and objective standards and methods while respecting local authorities, retention of high-quality staff in emergency departments, and having insufficient funding and resources to provide education and training opportunities

Describe Israel's Health Care System

Insurance coverage is mandatory for all Israeli citizens. Adults are required to pay a health tax. General tax revenues supplement the health tax revenues which the government distributes to the various health plans based on a capitation formula. Each year, the government determines how much from the general tax revenue should be contributed toward the NHI. Sickness funds offer a predefined basic package of health care services and are prohibited from discriminating against those who have preexisting conditions. Recently, mental health and dental care for children were added to the benefits package. Private health insurance also is sold to supplement the basic package.

Risk Rating

Insurance rating according to which high-risk individuals pay more than the average premium price, and low-risk individuals pay less than the average price

Balance Bill

Insurers started to limit reimbursement to a usual, customary, and reasonable amount. Each insurer determined its own what that charge should be through community or statewide surveys of what providers were charging. If the actual charges exceeded the UCR amount then providers would balance bill which asks the patients to pay the difference between the actual charges and the payments received from third party payers

Tertiary Prevention

Interventions to prevent complications from chronic conditions and prevent further illness, injury, or disability

Why is it important for hospital administrators to regard outpatient care as a key component of their overall business strategy?

It is important for hospital administrators to regard outpatient care as a key component of their overall business strategy because MCOs emphasize preventive and outpatient care now due to the fierce competition in the health care industry and the relentless drive to cut costs

Describe Japan's Health Care System

Japan has been providing universal coverage to its citizens through 2 main health insurance schemes: 1) an employer-based system modeled after Germany's SHI program and 2) a national health insurance program. Large employers have their own health programs through nearly 2,000 private, nonprofit health insurance societies . Smaller companies either band together to provide private health insurance or belong to a government-run plan. Day laborers, seamen, agricultural workers, the self-employed, and retirees are all covered under the national health care program. Individual employees pay 8% of their salaries as premiums and receive coverage for approximately 90% of the cost of medical services. Dependents receive less than 90% coverage. Coverage is comprehensive including most dental care and approved prescription drugs, and patients are free to select their providers. Providers are paid on a national fee-for-service basis set by the government and have little control over reimbursement

Chiropractors

Licensed practitioners who have completed the doctor of chiropractic degree (DC) degree. Requirements for licensure include completion of an accredited program that awards a DC degree and an examination by the state chiropractic board

Why is it important for health care managers and policymakers to understand the intricacies of the healthcare delivery system?

Managers need to know how to organize for the future, know how to deal with any threats to their institution's profitability and viability by transforming certain threats into new opportunities, know how to evaluate the implications of health policy and new reform proposals, know how to plan for the evolving trends and how to capture new health services markets from emerging trends, and and have to be able to comply with regulations set for their institution

Relative Value Units (RVUs)

Measures based on physicians' time, skill, and intensity required to provide a service

Crude Rates

Measures referring to the total population; they are not specific to any age groups or disease categories

Which particular factors that earlier may have been somewhat weak in bringing about national health insurance later led to the passage of Medicare and Medicaid?

Medicaid and Medicare were designed to cover only the most vulnerable populations. The proposals did not reengineer how the majority of the Americans would receive health care. The growing elderly population was becoming a politically active force among middle-class Americans.

Flat of the Curve

Medical care that produces relatively few or no benefits for the patient because of diminishing marginal returns

Medical technology encompasses more than just sophisticated equipment. Discuss

Medical technology includes health information system, managed care, and telemedicine. It includes life-saving procedures, such as bone marrow and organ transplants, as well as curative procedures, such as hip replacements

Describe how some of the changes in the health services delivery system have led to a decline in hospital inpatient days and a growth in ambulatory services

Medicare reimbursement and cost-saving efforts of managed care are the two main factors that have led to a decline in hospital inpatient days and a growth in ambulatory services. Medicare instituted the prospective payment system (PPS) for reimbursing hospitals in the mid-1980s. PPS reimbursement based on DRGs provides fixed case-based payment to hospitals. Hospitals, therefore, have a strong incentive to minimize the inpatient length of stay and continue treatment in an outpatient setting. The outpatient sector has fewer payment restrictions. Cost-containment strategies adopted by managed care also stress lower inpatient utilization, with a corresponding emphasis on outpatient services. These financial factors, for instance, have provided a major impetus for the unprecedented growth of home health care. Such changes, coupled with the availability of new technology, have also shifted a number of inpatient surgical procedures to the outpatient setting. Patients also have a strong preference for receiving health care in home and community-based settings and unless absolute necessary, most patients do not want to be institutionalized. Also, this allows people to have a strong sense of independence and control over their lives which are elements considered important for better quality of life. Traditionally, large hospitals were located in congested urban centers but increasing number of suburbanites now perceive these locations as inconvenient. Hence, many freestanding outpatient centers and satellite clinics operated by inner-city hospitals are now located in the suburbs

Enrollee

Member or individual covered under a health plan

What has been the main cause of the dichotomy between the way physical and mental health issues have traditionally been addressed by the health care delivery system?

Mental health is less objective than physical health. Physical function can be more readily observed while mental health requires assessment. Mental health is assessed by the presence of psychophysiological symptoms such as low energy, headache, and upset stomach and psychological symptoms such as nervousness, depression, and anxiety

Psychologists

Mental health professionals who must be licensed or certified to practice. These professionals may specialize in such areas as clinical, counseling, developmental, educational, engineering, personnel, experimental, industrial, psychometric, rehabilitation, school, and social psychology

Who are midlevel providers? What are their roles in the delivery of health care?

Midlevel providers (MLPs) are clinical professionals who practice in many areas similar to those in which physicians' practice but who do not have an MD or a DO practice. MLPs receive less advance training than physicians but more training than RNs. MLPs can often substitute for physicians. MLPs include physician assistants (PAs), NPs, and CNMs.

Emigration

Migration out of a defined geographic area

Immigration

Migration to a defined geographic area

Standards of Participation

Minimum quality standards established by the government regulatory agencies to certify providers for delivery of services to patients covered by Medicare or Medicaid

What are mobile health care services? Discuss the various types of mobile services

Mobile health care services are portable accessible health care services. Mobile health care services contains medical, diagnostic, and screening services. Ambulance service and first-aid treatment provided to the victims of severe illness, accidents, and disasters by trained emergency medical technicians (EMTs) are the most commonly encountered mobile medical services. Even specialized ambulance services, such as mobile coronary care units and shock-trauma vans and disaster relief vans, are staffed by paramedics and EMTs who have advanced training. In some situations, mobile eye care, podiatric care, and dental care units are an efficient and convenient means for providing certain type of routine health services. They are a convenient service for the elderly patients. Mobile diagnostic services include mammography and magnetic resonance imaging (MRI) units which can be taken to small towns and rural communities. They offer the advantages of convenience to patients and cost-efficiency in the delivery of diagnostic care. Health screening vans, staffed by volunteers who are trained professionals and operated by various non profit organizations, are often seen at malls and fair sites. Various types of health education and health promotion services and screening checks such as blood pressure and cholesterol screening, are commonly performed for anyone who walks in

M-Health

Mobile health; The use of wireless communication devices to support public health and clinical practice

Discuss, with particular reference to the roles of (a) organized medicine, (b) the middle class, and (c) American beliefs and values , why reform efforts to bring in national health insurance have historically been unsuccessful in the United States

Most middle class Americans value systems align with market justice. Individualism, self determination, and and distrust of government. The AMA was against one single health insurance plan. National health insurance became stigmatized as "socialized medicine" (any large scale government sponsored expansion of health insurance or intrusion in the private practice of medicine) AMA made it a goal to link national health care with communism

Describe the major types of health service professionals (physicians, nurses, dentists, pharmacists, physician assistants, nurse practitioners, certified nurse midwives) including their roles, training, practice requirements, and practice settings

NPs are individuals who have completed a program of study leading to competence as RNs in an extended role. Training for NPs covers topics of health promotion, disease prevention, health education, counseling, and disease management. NPs take health histories, provide physical exams and health assessments, and diagnose, treat, and manage patients with acute and chronic health conditions. The training of NPs may be a certificate program (9 months in duration) or a master's degree program (2 years of full-time study). States vary with regard to licensure and accreditation requirements for these roles. Most NPs are now trained in graduate or postgraduate nursing programs. In addition, NPs must complete clinical training in direct patient care. Certification examinations are offered by the American Nurses Credentialing Center, the American Academy of Nurse Practitioners, and specialty nursing organizations. NP specialties include pediatric, family, adult, psychiatric, and geriatric programs. NPs have statutory prescribing authority in almost all states, and they may serve as independent providers without supervision. NPs can also receive direct reimbursement as providers under the Medicaid and Medicare programs.

What is the difference between national health insurance (NHI) and a national health system (NHS)?

National health insurance is a tax-supported mechanism in which the government guarantees a basic package of health services to all citizens. In a national health system, in addition to financing a basic health package, the government also manages the infrastructure for the delivery of medical care

Registered Nurses (RNs)

Nurses who have completed an associate's degree (ADN), a diploma program, or a bachelor's degree (BSN) and are licensed to practice

Environment

One of the factors of the epidemiology triangle, which is external to the host; It includes the physical, social, cultural, and economic aspects of the environment

Agent

One of the factors of the epidemiology triangle, which must be present for an infectious disease to occur; In other words, an infectious disease cannot occur within an agent

Host

One of the factors of the epidemiology triangle; An organism, generally a human, who receives the agent and becomes sick

Virtual Physician Visits

Online clinical encounters between a patient and physician

Phantom Providers

Other providers who performed a service such as anesthesiologists, nurse anesthesiologists, pathologists, etc. bill for their services separately from the final bill

What is Medicare Part A? Discuss the financing and cost-sharing features of Medicare Part A. Which benefits does Part A cover? Which benefits are not covered?

Part A=the hospital insurance portion of Medicare, true entitlement program since people contribute during their working lives to Medicare through payroll taxes thus entitled to Part A benefits regardless of the amount of income and assets they may have, employer and employee share equally in financing , to qualify a person or person's spouse must have worked, earned a minimum specified amount, and paid Medicare taxes for at least 10 years to earn at least 40 credits but those who have earned less than 40 credits can get it by paying a monthly premium, covers inpatient services for acute-care hospitals,· psychiatric hospitals, inpatient rehabilitation facilities, skilled nursing facility (SNF) services, home health visits, and hospice care · Overview of benefits provided under Part A: 1) Maximum of 90 days of inpatient hospital care allowed per benefit period. Once 90 days used up, lifetime reserve of 60 days remain, 2) A total of 90 days of care per spell of illness is allowed for treatment in psych inpatient facility, with a 60-day lifetime reserve. Lifetime use limited to 190 days of treatment, 3) Medicare pays for up to 100 days of care in a Medicare-certified SNF, after inpatient hospitalization for at least 3 consecutive days, not including day of discharge. Admission to SNF must occur within 30 days of hospital discharge, 4) Medicare pays for home health care obtained from a Medicare-certified home health agency when a person is home-bound and requires intermittent or part-time skilled nursing care or rehabilitation. Payment is made for 60-days of care. A beneficiary can have unlimited days, and 5) For terminally ill patients, Medicare pays for care provided by a Medicare-certified hospice. A deductible applies to each benefit period except to home health and hospice and copayments based on the duration of services except for home health · Benefit period= a spell of illness beginning with hospitalization and ending when a beneficiary has not been an inpatient in a hospital or an SNF for 60 consecutive day, unlimited number of benefit period, applies to acute-care hospitals and inpatient rehabilitation facilities Noncovered services under Part A are long-term care, custodial services, and personal convenience services such as telephones, private-duty nurses, private rooms when not medically necessary

Discuss how the patient-centered medical home advances primary care.

Patient-Centered Medical Homes advance primary care by consisting of a team of doctors and other healthcare professionals who partner with patients and their families to deliver the best care possible by using a team approach, technology, and evidence-based practice. PCPs serve as advocates for patients to help them access services across the wide variety of health care services, ensuring that the patient's values, wishes, and directives are honored. PCMH typically focuses on greater use of preventive services, immunizations, and well-care visits

Fee For Service

Payment of separate fees to the providers for each separate service, such as examination, administering a test, and hospitalization

Preexisting Conditions

Physical and/or mental conditions that existed before the effective date of an insurance policy

Describe the major types of health service professionals (physicians, nurses, dentists, pharmacists, physician assistants, nurse practitioners, certified nurse midwives) including their roles, training, practice requirements, and practice settings

Physician Assistants (PAs) are defined as part of the health care team who work in a dependent relationship with a supervising physician to provide comprehensive care. PAs are licensed to perform medical procedures only under the supervision of a physician who may be on-site or off-site. Major services provided by PAs include evaluation, monitoring, diagnostics, therapeutics, counseling, and referral. In most states, PAs have the authority to prescribe medications. PA programs award bachelor's degrees, certificates, associate degrees, master's degrees, or doctoral degrees. The length of the program is 26 months.

Generalists

Physicians in family practice, general internal medicine, or general pediatrics

Specialists

Physicians who specialize in specific health care problems; Examples include anesthesiologists, cardiologists, and oncologists

Hospitalists

Physicians who specialize in the care of hospitalized patients

Medical Technology

Practical application of the scientific body of knowledge for the purpose of improving health and creating efficiencies in the delivery of health care

Defensive Medicine

Practitioners perform by prescribing additional diagnostic tests, scheduling return checkup visits, and maintaining copious documentation to protect themselves against the probability of litigation

Provider-Induced Demand

Practitioners who have a financial interest in additional treatments causing artificial demand; Functioning as patients' agents, physicians prescribe medical care beyond what is clinically necessary such as making more frequent follow-up appointments than necessary, prescribing excessive medical tests, or performing unnecessary surgery

What are the major distinctions between primary care and specialty care?

Primary care can be distinguished from specialty care based on time, focus, and scope of services provided to patients. Primary care is first-contact care and is regarded as the entry point to the health care system. Specialty care, when needed, generally follows primary care. Because specialty care is episodic, it is more focused and intense than primary care.

What are the main characteristics of primary care?

Primary care plays a central role in a health care delivery system. Compared to primary care, secondary, and tertiary care services are more complex and specialized. Primary care is distinguished from secondary and tertiary care according to its duration, frequency, and level of intensity. Primary care emphasizes prevention, diagnostic and therapeutic services, health education and counseling, and minor surgery. Primary care is the point of entry into the health services system where healthcare delivery is organized around primary care. Primary care is the first contact a patient makes with the health delivery system. Referrals for specialized services are made by primary care physicians. One of the main functions of primary care is to coordinate the delivery of health services between the patient and the myriad delivery components of the system. Hence, in addition to providing basic services, primary care professionals serve the role of patient advisor, advocate, and system gatekeeper. In this coordinating role, the provider refers patients to sources of specialized care, gives advice regarding various diagnoses and therapies, discusses treatment options, and provides continuing care over time. Coordination of an individual's total healthcare needs is meant to ensure continuity and comprehensiveness. Primary care is comprehensive because it addresses any health problem at any given stage of a patient's life cycle. Primary health care is regarded as essential health care. Primary care must consider the influence of the family on a patient's health status and be aware of the patient's living conditions, family dynamics, and cultural background. In addition, exemplary primary care requires an understanding of and a responsibility for the community's health

Voluntary Health Insurance

Private health insurance (in contrast to government-sponsored compulsory health insurance)

Dentists

Professionals who diagnose and treat dental problems related to the teeth, gums, and tissues of the mouth

Pharmacists

Professionals who have graduated from an accredited pharmacy program that awards a bachelor of pharmacy or doctor of pharmacy degree and have successfully completed a state board examination and supervised internship

Optometrists

Professionals who possess a doctor of optometry degree and have passed a written and clinical state board examination. An optometrist provide vision care-examination, diagnosis, and correction of vision disorders

Value

Provision of greater benefits or higher quality at the same or lower price levels (costs)

Categorical Programs

Public health care programs designed to benefit only a certain category of people

What are the main objectives of public health?

Public health is concerned with ensuring conditions that promote optimum health for society as a whole. Its main objectives are to prevent disease, prolong life, and promote health through organized community effort

Describe the scope of public health ambulatory services in the United States

Public health services in the US are typically provided by local health departments and the range of services offered varies greatly by locality. Generally, public health services are limited to well-baby care, sexually transmitted disease clinics, family planning services, screening and treatment for tuberculosis, and ambulatory mental health. Inner-city, poor, and uninsured populations are the main beneficiaries of these services. Health programs delivered in public schools fall under the public health domain and are limited to vision and hearing screening and assistance with dysfunctions that impede learning. Ambulatory clinics in prisons also fall in the public health domain

Why do patients sometimes use the hospital emergency department for nonurgent conditions? What are the consequences?

Reasons for nonurgent use of ED include unavailability of primary care, erroneous self-assessment of severity of ailment or injury, the 24-hour open-door policy, convenience, socioeconomic stress, psychiatric comorbidities, and a lack of social support (Hummel et al., 2014,;Liggins, 1993;Padgett and Brodsky, 1992). The uninsured and people on Medicaid use the most ED services. Many private physicians do not provide services to Medicaid enrollees because of low reimbursement which often leaves people on Medicaid without a regular source of primary care. Inappropriate use of emergency services wastes precious resources. Alternatives to the ED for nonurgent and routine care are critically needed- a problem that can be traced back to the US inadequate primary care infrastructure

Package Pricing

Refers to a bundled fee for a package of related services, EX: In the case of surgery, one all-inclusive price for surgeon's fees, hospital facilities, supplies, diagnostics, pathology, anesthesia, and postsurgical follow-up

Provider

Refers to any entity that delivers healthcare services and either independently bills for those services or is supported through tax revenues

Utilization

Refers to the quantity of health care consumed

Certified Nurse-Midwife (CNM)

Registered nurses with additional training from a nurse-midwifery program in areas such as maternal and fetal procedures, maternity and child nursing, and patient assessment. CNMs deliver babies, provide family planning education, and manage gynecologic and obstetric care. They can substitute for obstetricians/gynecologists in prenatal and postnatal care

What is the role of health risk appraisal in health promotion and disease prevention?

Risk factors associated with the host, agent, environment, and their health consequences are evaluated through a process called health risk appraisal. Only when the risk factors and their health consequences are known can interventions be developed to help individuals adopt healthier lifestyles. Interventions for counteracting the key risk factors include two main approaches:1) behavior modification geared toward the goal of adopting healthier lifestyles and 2) therapeutic interventions

Secondary Care

Routine hospitalization, routine surgery, and specialized outpatient care, such as consultation with specialists and rehabilitation. Compared to primary care, these services are usually brief and more complex, involving advanced diagnostic and therapeutic procedures

Why is there a geographic mal distribution of the physician labor force in the U.S.?

Rural areas lack an adequate supply of both PCPs and specialists even though patients in rural areas are sicker, older, and poorer than those in nonrural areas. Whereas 19.3% of US population lives in rural areas, only 11.4% of physicians practice there.

Home Health Care

Services such as nursing, therapy, and health-related homemaker or social services brought to patients in their own homes because such patients are generally unable to leave their homes safely to get the care they need

Palliation

Serving to relieve or alleviate, such as pharmacologic pain management and nausea relief

Experience Rating

Setting of insurance rates based on a group's actual health care expenses in a prior period. This allows healthier groups to pay less

Which measures have been or can be employed to overcome problems related to physician maldistribution and imbalance?

Several federal programs demonstrate success in increasing the supply of primary care services in rural areas by making scholarship support conditional on a commitment to future service in an underserved area.

Acute Condition

Short-term, intense medical care for an illness or injury usually requiring hospitalization

Morbidity

Sickness

Describe Singapore's Health Care System

Singapore has a system based on market competition and self-reliance. Singapore has achieved universal coverage through a policy that requires mandatory private contributions but little governmental financing. The program known as Medisave mandates every working person including self employed to deposit a portion of earnings in to an individual Medisave account. Employers are required to match employee contributions. These savings can be withdrawn only for 2 purposes: to pay for hospital services and some selected, expensive physician services, or to purchase a goverment-sponsored insurance plan called MediShield for catastrophic illness. For basic and routine services, people are expected to pay out of pocket. Those who can't afford to pay for healthcare services receive governmental assistance. ElderShield was created to defray out of pocket expenses for the elderly and severly disabled persons who require long-term care. The fee-for-service system of payment is widely used throughout Singapore. Patients can use their Medisave accounts or their family members' accounts to pay for outpatient services under the program

Social Resources

Social contacts that can be relied upon for support, such as family relatives, friends, neighbors, and members of a religious congregation. They are indicative of adequacy of social relationships

Allied Health Professional

Someone who has received a certificate; associate's, bachelor's, or master's degree; doctoral level preparation; or post-baccalaureate training in a science related to health care and has responsibility for the delivery of health or related services

Reinsurance

Stop-loss coverage that self-insured employers purchase to protect themselves against any potential risk of high losses

Durable Medical Equipment (DME)

Supplies and equipment not immediately consumed, such as ostomy supplies, wheelchairs, and oxygen tanks

Administrative Information Systems

Systems designed to assist in carrying out financial and administrative support activities such as payroll, patient accounting, materials management, and office automation

Clinical Information Systems

Systems that provide for organized processing, storage, and retrieval of information to support patient care processes

Summarize the government's role in technology diffusion.

Tech diffusion (new stuff comes in and used in practice) vs. tech imperative (latest thing) US has more high tech equipment than most other nations -but US lacks a central planning and utilization process to evaluate, limit and control costs; US also lacks a central budgeting process. Private insurers tend to follow Medicare decisions to determine which technology is covered under payment -Medicare reimbursement seeks "value" in reimbursement decisions -more generous insurance plans are associated with increased spending for new technology. The US govt does play a role in deciding which drugs, devices, and biologics are made available to Americans - they DO have control on regulation of drugs-Certificates of Need (to open a hospital), Research on Technology, Research Funding (funding independent researchers or agencies to develop new technologies)-focus on safety first, and then efficacy or effectiveness, and lastly cost of care

Synchronous Technology

Technology in which telecommunications occur in real time

Information Technology (IT)

Technology used for the transformation of data into useful information; it focuses on determining data needs, gathering appropriate data, storing and analyzing the data, and reporting the information generated in a user-friendly format

What is telemedicine? How do the synchronous and asynchronous forms of telemedicine differ in their applications?

Telemedicine= distance medicine, employs telecommunications technology for medical diagnosis and patient care when the provider and the client are separated by distance, and has applications in the delivery of specialized medical services. Synchronous Technology= allows telecommunication to occur in real time and allows a specialist at a distance to directly interview and examine a patient. Asynchronous Technology= employs store-and-forward technology that allows users to review the information later

Telephone Triage

Telephone access to a trained nurse for expert opinion and advice, especially during the hours when physicians' offices are closed

Briefly explain how a telephone triage system functions

Telephone triage is a type of access under managed care that is a means of bringing expert opinion and advice to the patient, especially during the hours when physicians' offices are closed. A telephone triage system operates 7 days a week, 24 hours a day in which the system is staffed by specially trained nurses who receive patients' calls. The nurses use a computer-based clinical decision support system so they can access the patient's medical history and view the most recent radiology and laboratory test results. The decision support system enables the nurse to give instructions on how to deal with the patient's problem. Consultation with a primary care physician is done when necessary. The nurse can direct patients to appropriate medical services, such as an ED or a physician's office

What provisions has the federal government made for providing health care to military personnel and to veterans of the US armed forces?

The Department of Defense has a Military Health System that provides medical services to active duty and retired members of the armed forces, their dependents, survivors, and former spouses. The program has also been extended to National Guard/Reserve members. TRICARE is the insurance arm of the military health system. Beneficiaries may obtain health care either through DOD's medical facilities or through services purchased from civilian providers. TRICARE offers several different health insurance plans including managed care and fee-for-service options and different options depending on whether the eligible beneficiaries lives in the US or overseas. For retirees over 65, TRICARE offers a plan that works with Medicare in which the enrollee must · enroll in Parts A and B. Service members who separate from service due to a service-connected injury or illness may be eligible for VA benefits and certain TRICARE benefits. TRICARE- and VA- eligible beneficiaries can choose to use either their TRICARE or VA benefits for each separate visits · Veterans' Health Administration= the health services branch of the VA, operates the largest integrated health services system in the US, this system encompasses more than 1700 patient clinics, community living centers, and other facilities, was originally established to treat veterans with war-related injuries and to help rehabilitate past service members with war-related disabilities, Congress requires VHA to provide services on a priority basis to veterans with service-connected illnesses and disabilities, low incomes, or special healthcare needs, funding comes from Congress, but suffer from capacity and financing constraints which results in lack of access and timely care for veterans

Provide a general overview of the Affordable Care Act. What is its main goal?

The Patient Protection and Affordable Care Act of 2010 (ACA) was created to reduce the number of uninsured people. Insurance companies were mandated to start covering children and young adults younger than age 26 under their parents' health insurance plans. The ACA required that all US citizens and legal residents must be covered by either public or private insurance. If they were not qualified for a public program, a health insurance marketplace allowed the individual to purchase a government-approved health plan offered by private insurers. Federal subsidies enabled low-income people to partially offset the cost of health insurance

Discuss the main cultural beliefs and values in American society that have influenced health care delivery and how they have shaped the health care delivery system

The US has a strong belief in the advancement of science and the application of scientific methods to medicine which increases the demand for the latest treatments and raise patients' expectations for finding cures but causes medical professionals to focus on clinical interventions not holistic aspects of health and use of alternative therapies. The US has been a champion of capitalism therefore health care has largely been viewed as an economic good (or service), not as a public resource. A culture of capitalism promotes entrepreneurial spirit and self-determination. Upper-tier access to health care services is available mainly through private health insurance. The culture of individualism emphasizes individual health rather than population health. Consequently, medical practice has been directed at keeping the individual healthy rather than the entire community. A concern for the most unprivileged classes in society led to creation of Medicaid and Medicare and Children's Health Insurance Program (CHIP). The US health care delivery is guided by principles of free enterprise and a general distrust of big government hence health care delivery is largely in private hands and a separation exists between public health functions and the private practice of medicine.

Describe the United Kingdom's Health Care System

The United Kingdom follow the NHS model. Its health delivery system is called the National Health Service. The system owns its hospitals and employs its hospital-based specialists and other staff on a salaried basis. The primary care physicians, referred to as general practitioners (GP), are mostly private practitioners. All NHS-insured patients are required to register with a local GP. The NHS emphasizes free point of access and equal access to all. Delivery of primary care occurs through primary care trusts (PCTs) in England, local health groups in Wales, health boards in Scotland, and primary care partnerships in Northern Ireland. PCTs have geographically assigned responsibility for community health services and each person living in a given geographical area is assigned to a particular PCT. PCTs function independently of the local health authorities and are governed by a consumer-dominated board and has its own budget allocations for primary care and hospital-based services. PCTs function like MCOs in the US

Surge Capacity

The ability of a healthcare facility or system to expand its operations to safely treat an abnormally large influx of patients

Universal Access

The ability of all citizens to obtain health care when needed

Access

The ability of an individual to obtain health care services when needed; Americans can access health care services if they have health insurance through their employers, are covered under a government health care program, can afford to buy insurance with their own private funds, are able to pay for services privately, or can obtain charity or subsidized care

Charge

The amount a provider bills for rendering a service

Reimbursement

The amount insurers pay to a provider and the payment may be just a portion of the actual charge

Need

The amount of medical care that medical experts believe a person should have to remain or become healthy

Health Informatics

The application of information science to improve the efficiency, accuracy, and reliability of health care services. Health informatics requires the use of information technology (IT), but goes beyond IT by emphasizing the improvement of health care delivery

Natality

The birth rate

Fertility

The capacity of a population to reproduce

Community-Oriented Primary Care (COPC)`

The combination of the elements of good primary care delivery with a population-based approach to identifying and addressing community health problems

Part A

The component of Medicare that provides coverage for hospital care and limited nursing home care

How can health care administrators and policymakers use the various measures of health status and service utilization? Please use examples to illustrate your answer

The conceptual approaches for defining health and its distribution help form a vision for the future and objective measures play a critical role in evaluating the success of programs and directing future planning activities

Health Plan

The contractual arrangement between the MCO (managed care organizations) and the enrollee including the collective array of covered health services that the enrollee is entitled to; The health plan uses selected providers from whom the enrollees can choose to receive services

Moral Hazard

The demand for health care greatly influences its financing. Health insurance increases the demand for covered services; the demand would be less if those same services were paid out of pocket. Increased demand means greater utilization of health services, given adequate supply. According to economic theory, insurance lowers the out of pocket cost of medical care to consumers; hence, they will consume more medical services than if they had to pay the entire price out of their own pockets. Consumer behavior that leads to a higher utilization of health care services when the services are covered by insurance is referred to as moral hazard (Feinstein, 1993)

Which factors are associated with the development of health services professionals in the US?

The development of these professionals is influenced by demographic trends, advances in research and technology, disease and illness trends, and the changing environment of health care financing and delivery

Health Risk Appraisal

The evaluation of risk factors and their health consequences for individuals

Environmental Health

The field that focuses on the environmental determinants of health

Discuss the two main factors that determine what should be an adequate mix of generalists and specialists

The first factor is how rigidly a health delivery system employs the concept of gatekeeping. The British national health service, for instance, employs this concept more rigidly than the Canadian health system. Consequently, the proportion of primary care physicians is 50 percent in Canada, compared to 70 percent in Britain. In the United States, gatekeeping is gaining prominence in the managed care system. Consequently, the proliferation of healthcare delivery through managed care is creating a greater demand for primary care physicians. The second factor driving the need for primary care providers is the propensity of people in a given population to utilize primary care services. Greater utilization suggests more professionals are needed to adequately serve a population. It is estimated that about 75%-85% of people in a general population require only primary care services in a given year. However, these proportions will vary in populations with special healthcare needs

Plan

The form in which health insurance, particularly private health insurance, is obtained. The plan specifies among other details, information pertaining to costs, covered services, and how to obtain health care when needed

Name the four basic functional components of the U.S. health care delivery system. Which role does each play in the delivery of health care?

The four basic components are financing, insurance, delivery, and payment. Financing pays for the purchase of health insurance. Insurance protects the buyers of health coverage against catastrophic risks. Delivery of health care enables people to receive services covered under their health insurance plans. Payment mechanisms allow providers to receive payments for services delivered to the insured

Cultural Authority

The general acceptance of professional judgement as valid. Physicians' cultural authority is reflected in the reliance placed on their evaluation of signs and symptoms, diagnosis of disease, and suggested prognosis

Migration

The geographic movement of populations between defined geographic units, which involves a permanent change of residence

Which main roles does the government play in the U.S. health services system?

The government is a major financier of health care delivery through the Medicare and Medicaid programs. The government determines eligibility criteria as to who can receive services under these programs; it also determines the reimbursement rates that providers will receive for rendering services to Medicaid and Medicare patients. In order to render services to Medicare and Medicaid, these organizations must comply with the standards of participation formulated by the government

Discuss the significance of an individual's quality of life from the health care delivery perspective

The implication is that desirable processes during medical treatment and successful outcomes should subsequently have a positive effect on an individual's ability to function, carry out social roles and obligations, and have a sense of fulfillment and self-worth

Insured

The individual who is covered for risk by insurance

Discuss the intermediary role of insurance in the delivery of health care

The insurance function introduces a third party into the transaction between the patient and the provider. Health insurance insulates the consumer from the cost of health care

Premium

The insurer's charge for insurance coverage; The price for an insurance plan

Who are the major players in the US health services system? What are the positive and negative effects of these players?

The key players in the system are the physicians, administrators of health service institutions, insurance executives, large employers, and the government. One positive effect of these opposing forces is that they prevent any single entity from dominating the system. On the other hand, they also make it difficult to achieve system wide reforms

Discuss the main ways in which current delivery of health care has become corporatized.

The main ways in which the current delivery of health care has become corporatized is by managed care, integrated health care services, advanced telecommunication, medical tourism, foreign dire investment in health services benefits foreign citizens, the creation of jobs overseas and medical care by the U.S. is in demand overseas

Discuss the definitions of health presented in this chapter in terms of their implications for the health care delivery system

The medical model defines health as the absence of illness or disease and implies that optimal health exists when a person is free of symptoms and does not require medical treatment, Medical sociologists define health as the state of optimal capacity of an individual to perform his or her expected social roles and tasks such as work, school, and household chores, The Society of Academic Emergency Medicine defines health as a state of physical and mental well being that facilitates the achievement of individual and societal goals, and The World Health Organization defines health as a state of complete physical, mental, and social well-being and not merely the absence of disease or infirmity. Health has so many different things it encompasses.

Discuss the relationship of dependency within the context of the medical profession's cultural and legitimized authority. What role did medical education reform play in galvanizing professional authority?

The medical profession's cultural authority is grounded in scientific knowledge, and its authority is legitimized when society accepts its competence in delivering specialized judgments. The profession's cultural authority is derived from the physician's superior knowledge and expertise in medicine. The patient becomes dependent on the medical profession's judgment and assistance. Advanced graduate medical education was instrumental in establishing the profession's cultural authority because it gave physicians a superior base of knowledge and skills. Once advanced graduate education became an integral part of medical training, it further legitimized the profession's authority and galvanized its sovereignty

Activities of Daily Living (ADLs)

The most commonly used measured of disability, which includes whether an individual needs assistance to perform basic activities, such as eating, bathing, dressing. toileting, and getting into or out of a bed or chair

Tertiary Care

The most complex level of care, which is typically institution based. highly specialized, and highly technological; Examples include burn treatment, transplantation, and coronary artery bypass surgery

Social Contacts

The number of activities a person engages in within a specified period of time, Ex: visits with friends and relatives, and attendance at social events, such as conferences, picnics, or other outings

Prevalence

The number of cases of a given disease in a given population at a certain point in time

Incidence

The number of new cases of a diseases in a defined population within a specified period

Medical Loss Ratio (MLR)

The percentage of premium revenue spent on medical expenses

Benefit Period

The period of illness beginning with hospitalization and ending when the beneficiary has not been an inpatient in a hospital or a skilled nursing facility for 60 consecutive days

Deductible

The portion of health care costs that the insured must first pay (generally up to an annual limit) before insurance payments kick in. Insurance payments may be further subject to copayment

Personal Health Expenditures

The portion of national health expenditures remaining after expenditures for research and construction, administrative expenses incurred in health insurance programs, and costs of government public health activities are subtracted. These expenditures are for services and goods related directly to patient care

Risk

The possibility of a substantial financial loss from an event for which the probability of occurrence is relatively small

Self-referral

The practice in which physicians order services from laboratories or other medical facilities in which they have a direct financial interest, usually without disclosing this conflict of interest to the patient

Balance Bill

The practice in which the provider bills the patient for the leftover sum after insurance has only partially paid the charge initially billed

Comorbidity

The presence of more than one health problem in an individual

Rate

The price for a health care service set by a third-party payer

Briefly describe the concepts of market justice and social justice. In which ways do the two principles complement each other and in which ways are they in conflict in the U.S. system of health care delivery?

The principle of market justice leaves the fair distribution of health care up to the market forces in a free economy. Medical care and its benefits are distributed based on people's willingness and ability to pay. The free market implies that giving people something they have not earned would be morally and economically wrong. The principle of social justice is when equitable distribution of health care is a societal responsibility which is best achieved by letting the government take over the production and distribution of health care. Social justice regards health care as a social good rather than an economic good that should be collectively financed and available to all citizens regardless of the individual recipient's ability to pay. The two principles complement each other in the employer-based health insurance available to most middle-class working Americans and the publicly financed Medicare, Medicaid, and CHIP coverage for certain disadvantaged groups. Insured populations access health care services delivered mainly by private practitioners and private institutions and Tax-supported county and city hospitals, public health clinics, and community health centers can be accessed by the uninsured in areas where such services are available. Market and social justice principles creates conflicts when health care resources are not uniformly distributed throughout the US and when there is a general shortage of primary care physicians. Consequently, many Medicaid-covered patients have difficulty obtaining timely access, particularly in rural and inner-city areas. This conflict is partly created by artificially low reimbursement from public programs while reimbursement from private payers is more generous

Technology Diffusion

The proliferation of technology once it is developed


Kaugnay na mga set ng pag-aaral

choose the word that best completes the sentence

View Set

Biomechanics final exam quiz questions

View Set

BioPsychology - All Chapters - Final Exam

View Set

Ap Euro all multiple choice questions

View Set